Sei sulla pagina 1di 102

For

3th year & 4th year


students

MCQs in
Oral and Maxillofacial

Surgery

Faculty of dentistry
Minia University
2017-2018
Page 1
TopicsIndex No

• GENERAL PRINCIPLES OF ORAL SURGERY 3


• INSTRUMENTS 9
• GENERAL ANESTHESIA 13
• LOCAL ANESTHESIA 16
• EXODONTIA 33
• FACIAL SPACE INFECTIONS 46
• BIOPSY PROCEDURES 53
• TEMPORO-MANDIBULAR JOINT 55
• MAXILLARY SINUS 58
• CYSTS AND TUMOURS 60
• SALIVARY GLAND 64
• PREPROSTHETIC SURGERIES 67
• IMPLANTS AND MISCELLANEOUS 70
• TRAUMATOLOGY 73
• CARDIO-PULMONARY RESUSCITATION 93
• MISCELLANEOUS 99

Edited by/
Abdelrahman Tarek Mohamed

Page 2
GENERAL PRINCIPLES OF ORAL SURGERY

1. Which one of the following is a contraindication for extraction of teeth


a) Endocarditis b) Vincent’s infection
c) Glomerulonephritis d) Rheumatic fever
Ans: B
2. Antibiotic prophylaxis dose of clindamycin in oral surgical procedures for
infective endocarditis is
a) 600 mg 1 hr before procedure b) 300 mg 1 hr before procedure
c) 600 mg 2 hr before procedure d) 300 mg 2 hr before procedure
Ans: A
3. Prophylactic antibiotic coverage before dental extraction is indicated for
patients with each of the following conditions except
a) Kidney damage needing hemodialysis b) Prosthetic aortic valve
c) Rheumatic heart disease d) Coronary artery bypass
Ans: D
4. The best time of extraction in pregnancy is
a) First trimester b) Second trimester
c) Third trimester d) None of these
Ans: B
5. During the tooth extraction, a known cardiac patient experienced angina.
This was most likely precipitated by
a) allergy to the anesthetic agent b) the stimulus of pain and anticipation
c) adrenaline in the drug d) the upright position in the chair
Ans: B
6. For a patient undergoing anticoagulant therapy, who requires unavoidable
dental surgical treatment, the dental surgeon should
a) not take up the patient till the therapy is over
b) bring down the dosage and proceed further
c) adjust the dosage before and after dental visit
d) consult the physician of the patient for joint decision
Ans: D
7. A 60-year-old male with previous h/o of myocardial infarction is on 75 mg
aspirin daily for the l< 6 months. He has to undergo extraction of carious
lower 6. The indicated protocol treatment is
a) Extraction of the teeth without any change in the dose schedule of aspirin
b) Discontinue aspirin atleast for 2-3 days before extraction
c) Discontinue aspirin atleast for a week before extraction
d) Give DDAVP infusion to counteract the effects of aspirin and proceed with extraction
Ans: A

Page 3
8. The following history and laboratory value contraindicate extraction of
mandibular first molar
a) Hb% is 15 gm% and hematocrit value is 42 per cent
b) Five months' pregnant lady without any complications
c) Rheumatic patient taking penicillin
d) Prothrombin time is 45 seconds
Ans: D
9. Patient who is receiving anticoagulants requires extraction of a tooth.
Laboratory data indicating prothrombin time is 21 seconds with a control time
of 15 seconds. The treatment should consist of
a) regulation of the anticoagulant level with heparin
b) injection of vit K and immediate extraction
c) injection of vit K and extraction after half an hour
d) extraction of the tooth and use of sutures and local hemostatic agent
Ans: D
10. ‘Rubber band’ extraction is a method of extraction in patients having
a) bleeding disorders b) myocardial infarction and angina pectoris
c) supernumerary teeth d) impacted teeth
Ans: A
11. A male is taking 60 mg of hydrocortisone daily, on the day of extraction,
this dose should be
a) remain unchanged b) reduced to half
c) doubled d) reduced to one fourth
Ans: A
12. The patients who are deficient of endogenous steroids and undergoing
maxillofacial surgery, the most preferred full spectrum corticosteroid for pre,
intra and post operative prophylactic steroid therapy is
a) Dexamethasone b) Methyl prednisolone
c) Triamcinolone d) Hydrocortisone
Ans: D
13. After the surgical removal of chronically infected teeth in a controlled
diabetic patient, which of the following is of utmost importance
a) Post operative check of patient for bleeding
b) Return of patient to diet of a normal individual
c) Re-evaluation of insulin dosage being taken by the patient
d) Medication for marked post operative pain and discomfort
Ans: C

Page 4
14. A 48 years old diabetic patient wants extraction of 2 teeth which are in
grade I mobility. He takes 40 units of NPH insulin daily. Which of the
following management procedure in indicated to re-move the teeth
a) Defer treatment, let him take breakfast, but not insulin to avoid
hypogiycemia.
b) Defer treatment, let the patient take his insulin but tell him to avoid
breakfast.
c) Treat the patient in the usual manner without much alteration in the diet and
drugs
d) Admit the patient in a hospital for the procedure.
Ans: C
15. A patient who is a grand mal epileptic requires emergency extraction. One
should
a) advise the patient that procedure requires admission to the hospital.
b) proceed with extraction using xylocaine without adrenaline.
c) proceed with extraction if anti-convulsive medication is employed by
patient.
d) inform the patient that in his condition no extraction should be performed.
Ans: C
16. The blood product of choice for treatment of hemophilics is
a) fresh blood b) cryoprecipitate
c) fresh frozen plasma d) plasma
Ans: B
17. The recommended time interval between the radiation and surgery to
avoid Osteoradionecrosis is
a) 4-6 weeks after radiotherapy b) 6-12 months after radiotherapy
c) Immediately before radiotherapy d) Any time during the course of radiotherapy
Ans: A
18. An absolute contraindication for extraction of teeth is:
a) hypertension b) myocardial infarction
c) thyrotoxicosis d) central hemangioma
Ans: D
19. The antibiotic cover is mandatory before extraction in the following
condition of the heart
a) Ischemic heart disease (IHD) b) Hypertension
c) Congestive cardiac failure. d) Congenital heart disease.
Ans: D
20. Drug of choice for parenteral administration in patients allergic to penicillin
for prophylaxis againstinfective endocarditis is?
a) Ciprofloxacin b) Gentamycin
c) Clindamycin d) Cefotaxime Ans: C

Page 5
21. Which of the following medication is required preoperatively for a patient
with mitral valve replacement
a) Ampicillin b) Cloxacillin
c) Benzathine penicillin d) Procaine penicillin
Ans: D
22. A pregnant patient in second trimester falls into syncope during extraction
of upper molars. She should be kept in
a) trendelenburg position b) head down towards her feet
c) right lateral position d) left lateral position
Ans: D
23. 30-year-old female patient with B.P. of 150/80 mm Hg requires treatment.
You would
a) Recommend general anesthesia b) Seek medical treatment to lower BP
c) Refer the patient to an oral surgeon d) Use sedation preoperatively
Ans: D
24. Elective dental extractions and minor oral surgical procedures on a patient
who has had a myocardial infarct two months' prior, are best
a) performed using an adrenaline free local anaesthetic, because of adrenaline
action on heart muscle.
b) performed under oral diazepam,
c) postponed until at least six months have elapsed.
d) performed with normal.
Ans: C
25. A 50 years old patient on oral anticoagulants for cardiac illness, requires
tooth extraction. INR value is 1.5, dentist should?
a) proceed for dental extraction
b) anticoagulants should be stopped before dental extraction.
c) oral anticoagulants should be stopped and heparin started.
d) additional tests are required.
Ans: A
26. No surgical procedure should be performed in patients with
thrombocytopenia unless the platelet count is
a) >60,000 b) >70,000
c) >80,000 d) >50,000
Ans: C
27. In a patient of liver disease the possible complication during extraction is
a) dry socket b) fascial space infection
c) bleeding d) loss of clot
Ans: C

Page 6
28. A 45 years old man who is on dicoumoral therapy requires tooth
extraction. His medical history indicated myocardial Infarction two years ago.
This patient is receiving anticoagulant drugs. The dentist should
a) never operate on this patient
b) avoid performing surgery until the anticoagulant had been discontinued for
at least three days
c) perform surgery if prothrombin time is 18 seconds
d) advice vitamin K to avoid post operative haemorrhage
Ans: C
29. Among the following which condition will require specific management
prior to emergency extraction?
a) Hemophilia b) Glaucoma
c) Carcinoma tongue d) Mixed tumor of parotid
Ans: A
30. A 45 yr old woman insists on extraction of painful mandibular first molar.
She has not rested for 2 days and nights because of pain. Her medical history
reveals she took 30mg of prednisone daily for one year but stopped taking the
medication three months ago. The dentist should
a) relieve the symptoms by giving antibiotics and analgesics.
b) give steroid supplements and extract the tooth under local anesthesia.
c) avoid procaine anesthetic.
d) extract the tooth under local anesthesia in a usual manner.
Ans: B
31. A patient is under steroid therapy and needs extraction of chronically
infected teeth. Premediction includes
a) Antihypertensive drugs to prevent shock
b) Atropine to prevent cardiac arrest by vagal stimulation
c) Antihistamines d) Antibiotics
Ans: D
32. Hypoglycemia may occur in a patient taking insulin and undergoing
extraction when
a) extraction is done on empty stomach
b) patient had an infection
c) patient had no exercise in the morning
d) patient consumed breakfast before extraction
Ans: A

Page 7
33. Patients with end-stage renal disease are of increased risk when undergoing
extraction of teeth because they
a) have an increased tendency to bleed
b) are susceptible to infections
c) are often on steroid therapy
d) All of the above
Ans: D
34. A patient is on periodic renal dialysis. Minor oral surgery should be
performed
a) one day before dialysis
b) on the day of dialysis
c) one day after dialysis
d) one week after dialysis
Ans: C
35. Incidence of osteoradionecrosis is increased in following cases
a) Extraction following radiotherapy.
b) Leukaemic patients
c) Patients on hyperbaric oxygen therapy
d) Xerostomia
Ans: A
36. In a patient with maxillofacial injury narcotics like morphine are
contraindicated because
a) Morphine causes myosis b) Morphine causes mydriasis
c) Morphine is respiratory irritant d) Morphine is circulatory depressant
Ans: A

Page 8
INSTRUMENTS

1. A straight elevator is properly used to advantage when the


a) adjacent tooth is the fulcrum b) tooth is isolated
c) interdental bone is fulcrum d) adjacent tooth is not to be extracted
Ans: C
2. Most of the elevators used in exodontias works on the principle
a) Class I lever b) Class II lever
c) Class III lever d) Wheel and axle
Ans: A
3. Advantage of chisel and mallet over bur in removing bone is
a) less skill is needed b) psychologically more acceptable
c) no coolant is necessary
d) chisel & mallet are good instruments to remove dense bone
Ans: C
4. The tips of anatomic forceps should be placed
a) on the crown portion b) at the junction of clinical and anatomical crowns
c) near junction of apical and middle thirds of clinical crown
d) on the root surface as far apicaily as possible
Ans: D
5. Rongeur is commonly used to
a) cut the bone b) extract third molars
c) place the sutures in the areas where access is limited
d) None of the above
Ans: A
6. To luxate a tooth with a forceps the movements should be
a) firm and deliberate primarily to the facial surface with secondary movements
to the lingual surface
b) sharp and definite so periodontal ligament tears easily
c) gentle wiggles so patient get used to the pressure
d) figure of “eight” motion
Ans: A
7. The scalpel blade most commonly used for oral surgical procedures is
a) No: 22 b) No: 12
c) No: 11 d) No: 15
Ans: D
8. Osteotome is
a) monobevelled b) bibevelled
c) tetrabevelled d) not contain any bevels
Ans: B

Page 9
9. The direction of the bevel of the chisel during bone cutting is
a) away from the bone to be sacrificed
b) towards the bone to be sacrificed
c) independent of the bone to be sacrificed
d) parallel to stress lines
Ans: B
10. In endodontic surgery, which of the following about chromic gut suture is
not true……
a) Its properties are inferior to plain gut suture
b) It is coated with chromium trioxide
c) It is less bio compatible than plain gut suture
d) Its absorption is faster than the plain gut sutures
Ans: D
11. Polyglycolic acid suture material (“Vicryl”) is a/an
a) absorbable natural suture b) absorbable synthetic suture
c) non-absorbable natural suture d) non-absorbable synthetic suture
Ans: B
12. The suture material most commonly used for intra oral wound closure is
a) Vicryl b) Silk c) Nylon d) Catgut
Ans: B
13. Among the following suture materials which one elicits more tissue reaction
a) Catgut b) Silk c) Nylon d) Linen
Ans: A
14. Which of the following is commonly used as preservative for gut sutures
a) Ethyl alcohol b) Isotonic saline
c) Hypertonic saline d) Isopropyl alcohol
Ans: D
15. Absorbable suture is
a) Vicryl b) Silk c) Nylon d) Prolene
Ans: A
16. How much is the concentration of chlorhexidine used for rapid hand
disinfection prior to any surgical procedure
a) 2% b) 5% c) 15% d) 20%
Ans: D
17. The elevator can be used to advantage when
a) the tooth to be extracted is isolated
b) the interdental bone is used as a fulcrum
c) the adjacent tooth is not to be extracted
d) multiple adjacent teeth are to be extracted
Ans: D

Page
10
18. The best example of an elevator which works on wheel and axle principle
a) Howarth’s periosteal elevator b) Winter cross bar elevator
c) Millers apex elevator d) None of the above
Ans: B
19. The elevators that employ both the lever and wedge principles during
extraction of root tip are
a)Pott’s elevators and apex elevators b)Apex elevators and straight elevator
c)Straight elevators and pott’s elevators d)Pott’s elevators and cryer elevators
Ans: B
20. Number 16 cowhorn forceps are specially designed to extract
a) maxillary first and second premoiars b) mandibular central incisors
c) mandibular molars d) maxillary molars
Ans: C
21. One decided to use forceps for removal of a tooth. The first direction for
the force to be applied is
a) buccally b) lingually or palatally
c) apically d) occlusally
Ans: C
22. To extract a tooth the following type of grip is most desirable
a) The whole of the inner surface of the forceps blade should fit the root surface
b) Only edges of the forceps blade should contact the tooth (two-point contact)
c) Only single lever contact between forceps blade and root surface (one-point
contact)
d) Grip plays little role in extraction
Ans: A
23. A dermatome is used
a) to remove scar tissue b) to harvest skin grafts
c)to abrade skin which is pigmented d) for pairing of lacerated soft tissue
Ans: B
24. The prime purpose of Bibevel chisel is
a) split teeth b) sharpen the angles
c) remove bone d) engage point establishment
Ans: A
25. Prolene (polypropelene) suture is
a) synthetic absorbable b) natural absorbabale
c) synthetic non-absorbable d) synthetic absorbable
Ans: C
26. The following is one of the advantages of chromic gut over plain gut suture
a) Greater ease of use b) Non-absorbability
c) Greater strength d) Less expensive
Ans: C

Page
11
27. The following suture material has the memory property
a) Silk b) Catgut c) Nylon d) Linen
Ans: C
28. Which of the following is non-resorbable suture material?
a) Dacron b) Catgut
c) Polyglycolic acid d) Polyglactin 910
Ans: A
29. Gut suturing material is absorbed by
a) Oxidation b) Hydrolysis
c) Fibrinolysis d) Proteolysis
Ans: D
30. Which suture material has maximum life of 210 days?
a) Vicryl b) Chromic catgut
c) Dexon d) Polydiaxonone
Ans: D

Page
12
GENERAL ANESTHESIA

1. The patient’s ability to maintain an independent airway, and respond


appreciably to physical stimulation and verbal commands is seen in
a) general anesthesia b) deep sedation
c) conscious sedation d) deep sedation and general anesthesia
Ans: C
2. Sedation by which of the following routes can be reversed rapidly?
a) Intravenous b) Oral
c) Inhalation d) Intramuscular
Ans: C
3. Of the following in which condition general anaesthesia is contraindicated?
a) Hemoglobinopathy b) Allergy
c) Diabetes mellitus d) All of the above
Ans: A
4. Among the following in which patient GA is contraindicated?
a) An adult male over 65 years of age
b) Patient with acute respiratory infection
c) Child under 6 years of age
d) Patient with wheel compensated heart disease
Ans: B
5. Three common symptoms indicating the correct level of sedation after
diazepam administration is
a) Blurring of vision, slurring of speech & loss of GAG reflex
b) 50% ptosis of eyelids, blurring of vision, slurring of speech
c) Paresthesia of lips, tongue and fingers
d) Sweating over the face, itching of the nose and loss of gag reflex
Ans: B
6. Level of analgesia is best monitored by
a) eye movements and absence of corneal reflex
b) eye movements and absence of conjunctival reflex
c) respiration, rate, rhythm and type of respiratory movements
d) verbral response
Ans: D
7. In case of conscious sedation for children aged 6 months to 3 years, clear
liquids should be stopped
a) 4 hrs before the procedure b) 6 hrs before the procedure
c) 8 hrs before the procedure d) 10 hrs before the procedure
Ans: A

Page
13
8. In Conscious Sedation Nitrous Oxide and Oxygen are used in proportion of
a) 60% N2O & 80% O2 b) 20% N2O & 30% O2
c) 80% N2O & 20% O2 d) 40% N2O & 60% O2
Ans: D
9. Nitrous oxide is
a) a good analgesic but a poor anesthetic
b) a good anesthetic but a poor analgesic
c) a good analgesic and a good anesthetic
d) a poor analgesic and a poor anesthetic
Ans: A
10. Following administration, nitrous oxide will become saturated in blood and
dissolve in serum fraction of blood within?
a) 3-5 mins b)10 mins c)10-15 mins d)6-8 min
Ans: A
11. Which of the following side effects is seen commonly with the
administration of nitrous oxide and oxygen
a) Hallucinations and dreams b) Tachycardia
c) Tremors d) Nausea
Ans: D
15. The colour of a nitrous oxide cylinder is
a) red b) blue c) white d) black
Ans: B
16. Preferable patient position in recovery room after ambulatory general
anesthesia is
a) supine b) sitting
c) prone d) lateral
Ans: D
17. Compared to inhalation, IV sedation
a) produces more predictable amnesia
b) may produce more serious complications
c) may result in deeper sedation
d) All of the above are correct
Ans: D
18. The following two laboratory tests should be considered before deciding
whether to use general anesthesia or not
a) Total WBC count and prothrombin time
b) Total WBC count and urine analysis
c) Bleeding time and clotting time
d) Complete blood count and urine analysis
Ans: D

Page
14
19. Which of the following symptoms is seen in a patient administered with 20-
40% nitrous oxide
a) Paresthesia b) Floating sensation c) Sweating d) None of the above
Ans: B
20. In dentistry which sedatives are generally used:
a) Benzodiazepines b) Morphine c) NSAID d) Pethidine
Ans: A
21. Only nitrogen oxide alone is not used as a general anaesthetic agent
because of the
a) poor analgesic properties
b) expense of the agent and its explosive hazard
c) difficulty in maintaining an adequate oxygen concentration
d) hepatotoxic nature
Ans: C
22. In which conditions N2O-O2 sedation in contraindicated?
a) Emphysema b) Upper respiratory obstruction
c) Emotional instability d) All of the above
Ans: D
23. The Color of oxygen cylinder is?
a) Blue b) White c) Black d) Yellow
Ans: C
24. Of the following which is most common postoperative complication of out
patient general anaesthesia
a) Pneumonia b) Lung abscess
c) Nausea d) Atelectasis
Ans: C
25. After the general anesthesia, vomiting may occur. The immediate post
operative complication following aspiration of liquid vomitus into trachea and
bronchi is
a) lung abscess and tuberculosis
b) bronchitis and chemical pneumonia
c) pleurisy and atelectasis
d) pleurisy and empyema
Ans: B
26. Which of the following respiratory conditions is most alarming during
patient sedation in dental clinic?
a) Apnea b) Dyspnea c) Hyperapnea d) Tachyopnea
Ans: A

Page
15
LOCAL ANESTHESIA
1.The most rigid portion of a syringe is
a) hub b) shaft c) needle d) handle
Ans: A
2. According of pain, large fiber impulses tend to
a) by pass the central control process
b) inhibit the effect of painful stimuli transmitted by small fibers
c) transmit severe chronic pain only
d) potentiate the effect of painful stimuli
Ans: B
3. Which of the following is correctly matched. LA substance according to
biological site and mode of action.
LA substances Class
i- Bio toxin 1. class A
ii- Quaternary ammonium analogue 2. class B
iii- Benzocaine 3. class C
iv- Articaine 4. class D
a) (i-4), (ii-2), (iii-1), (iv-3) b) (i-4), (ii-3), (iii-2), (iv-1)
c) (i-2), (ii-1), (iii-3), (iv-4) d) (i-1), (ii-2), (iii-3), (iv-4)
Ans: D
4. The primary site of action of the local anesthetic is on
a) axoplasm b) epineurium
c) nerve membrane d) perineurium
Ans: C
5. Non-myelinated axons differ from myelinated ones in that they
a) are more excitable b) lack nodes of ranvier
c) are not capable of regeneration
d) are not associated with schwann cells
Ans: B
6. Rapid onset of action seen by local anesthesia in small nerve endings is due
to
a) increased threshold of small nerves due to depolarization
b) low pH of small nerve fibres
c) high ratio of surface area to the volume of small nerve fibres
d) increased resting potential of small nerve fibres
Ans: C
7. To give feld block the LA should be deposited near
a) main trunk b) large branch of peripheral nerve
c) small nerve endings d) periodontal ligament
Ans: B

Page
16
8. A contraindication to local infiltration technique
a) Hypertension b) Diabetic patient with presence of urine sugar
c) Infection in the arch d) Kidney problems
Ans: C
9. For extraction of mandibular molar, anesthesia is given to act on:
a) inferior alveolar nerve b) buccal nerve
c) lingual nerve d) masseteric nerve
Ans: A
10. During inferior alveolar nerve block the LA solution is deposited into which
space
a) Infratemporal space b) Sub maxillary space
c) Pterygo mandibular space d) Massetric space
Ans: C
11. In case of Gow-Gates technique the target area is
a) Neck of condyle b) Head of the condyle
c) Medial side of the ramus d) Lateral side of the condyle
Ans: A
12. In Vazirani-Akinosis mandibular block which is not a last mark
a) Mucogingival fold in relation of maxillary third molar
b) Maxillary tuberosity
c) Soft tissue medial ramus d) Coronoid notch
Ans: C
13. High tuberosity technique is a technique used to anestetize
a) maxillary nerve b) posterior superior alveolar nerve
c) mandibular nerve d) trigeminal nerve trunk
Ans: A
14. For extraoral maxillary nerve block the target area is
a) anterior to lateral pterygoid plate b) posterior to lateral pterygoid plate
c) pterygomandibular fossa d) pterygomandibular fissure
Ans: A
15. Mental foramen opening is directed
a) forward and medial direction b) backward and medial direction
c) forward and lateral direction d) backward and lateral direction
Ans: D
16. Mucous membrane on the buccal aspect of the upper alveolar process is
innervated by buccal nerve and?
a) superior alveolar nerve b) mental nerve
c) interior alveolar nerve d) greater palatine nerve
Ans: A

Page
17
17. Anterior palatine nerve is anesthetized by anterior palatine nerve block. The
subjective symptoms include
a) feeling of numbness in the posterior palate
b) feeling of numbness in the anterior part of the palate near the incisors
c) feeling of numbness of whole palate
d) little or no subjective symptoms
Ans: A
18. Greater palatine foramen is situated
a) between first and second maxillary molars
b) between second and third maxillary molars
c) between two central incisors
d) between first and second maxillary premolars
Ans: B
19. The following deep bony landmark is important in performing a block of
the II and III divisions of the trigeminal nerve from the lateral approach
a) Temporal surface of sphenoid b) Perpendicular plate of the palatine bone
c) Lateral plate of the pterygoid d) Styloid process
Ans: C
20. After the posterior superior nerve block patient should experience
a) numbness of the lip b) numbness of the palate till the first premolar
c) few symptoms of anesthesia d) numbness of that side of face
Ans: C
21. The sudden appearance of a “Dumb-bell” swelling after the administration
of PSA block is due to penetration in:
a) Maxillary artery b) Pterygoid plexus of veins
c) reater palatine artery d) Sphenopalatine artery
Ans: B
22. A surgical procedure required bilateral block. A bilateral inferior alveolar
nerve block is
a) not contraindicated b) should rarely be performed
c) may cause space infection
d) dangerous because patient may swallow the tongue
Ans: A
23. The following mandibular teeth can be removed without pain after the
inferior alveolar and lingual nerve block
a) All molars and second premolars b) First molar and premolars
c) Canine and first premolar d) All teeth in that quadrant
Ans: C

Page
18
24. In the initial stage of paralysis of facial nerve
a) Tongue deviate to the same side on protrusion
b) Tongue deviate to the opposte side on protrusion
c) No deviation of the tongue d) None of the above
Ans: A
25. Which of the following is true regarding local anesthesia?
a) They are basic salts of weak acids b) Not effective in alkaline pH
c) Form salts with acids d) They are acidic salts of weak bases
Ans: D
26. It is difficult to obtain local infiltration anesthesia in the presence of
inflammation/infection because of
a) a decreased PH b) increased vascularity
c) oedema d) pain
Ans: A
27. Alkalization of LA
a) decreases the onset of action b) speeds the onset of action
c) increases the efficiency of LA d) Both B and C
Ans: D
28. Lidocaine is an example of an
a) Acid b) Amide c) Ester d) Aldehyde
Ans: B
29. A patient is allergic to PABA. Most probably he will also be allergic to
a) lignocaine b) carbocaine
c) prilocaine d) tetracaine
Ans: D
30. The activity of procaine is terminated by
a) elimination by kidney b) storage in the adipose tissue
c) metabolism in the liver only
d) metabolism in the liver and by pseudocholinesterase in the plasma
Ans: D
31. The longest acting, most potent and most toxic L.A. is
a) lidocaine b) dibucaine c) bupivacaine d) tetracaine
Ans: B
32. Sodium bisulphate used in dental local anesthetic acts as
a) antibacterial for lignocaine b) antibacterial for adrenaline
c) antioxidant for methylparaben d) antioxidant for adrenaline
Ans: D
33. Xylocaine strength used in dentistry is
a) 2% b) 5% c) 8% d) 10%
Ans: A

Page
19
34. Alpha adrenergic agonists are used in combination with local anesthetics to
a) increase the rate of liver metabolism of local anesthetic
b) increase the concentration of L.A. at receptor site
c) stimulate myocardial contraction d) increases vascular absorption of L.A
Ans: B
35. Adrenaline is not added to which cases with local anesthetics
a) Spinal anesthesia b) Infiltration anesthesia
c) Surface anesthesia d) Ring block
Ans: D
36. Among the following items in the past medical history, which would most
affect the choice of local anesthetic agent?
a) RHD b) Hyperthyroidism
c) Adrenal insufficiency d) Asthma
Ans: B
37. The onset of action of lidecaine is
a) 1-2 minutes b) 5-10 minutes c) 3-5 minutes d) 7-8 minutes
Ans: C
38. Which of the following local anesthetic is a vasoconstrictor
a) Lidocaine b) Procaine c) Bupivacaine d) Ropivacaine
Ans: D
39. Which of the following is not used as a topical LA?
a) Procaine b) Tetracaine c) Lidocaine d) Benzocaine
Ans: A
40. Maximum available and effective con-centration of LA for topical use is
a) 2% b) 5% c) 10% d) 15%
Ans: B
41. Longest acting local anesthesia is
a) Bupivacaine b) Tetracaine c) Lidocaine d) Procaine
Ans: A
42. Pka of lignocaine is
a) 7.2 b) 7.4 c) 7.6 d) 7.9
Ans: D
43. The following is least toxic of all local anesthetics
a) Tetracaine b) Mepivacaine c) Prilocaine d) Chloroprocaine
Ans: D
44. Carticaine is the other name for
a) Articaine b) Mepivacaine c) Procaine d) Lidocaine
Ans: A

Page
20
45. Maximum recommended dosage of lignocaine with 1:1,00,000 adrenaline
subcutaneously is
a) 100 mg b) 30 mg c) 500 mg d) 700 mg
Ans: C
46. The maximum dose of lignocaine without adrenaline that can be admitted
to a patient is
a) 4 mg/kg body wt b)5 mg/kg body wt
c) 7 mg/kg body wt d) 9 mg/kg body wt
Ans: A
47. How many mg of 2% lidocaine present in the cartidge?
a) 36 mg b) 18 mg c) 180 mg d) 360 mg
Ans: A
48. How many mg of adrenaline is in each cartridge of 2% lignocaine with
1:1,00,000 adrenaline
a) 0.36 mg b) 0.036 mg c) 0.018 mg d) 0.18 mg
Ans: C
49. How much lignocaine is present in 2.0 ml of 2% lignocaine solution?
a) 40 mg b) 20 mg c) 30 mg d) 2 mg
Ans: A
50. Which of the following local anesthetic will be suitale for a hypertensive
patient suffering from a heart disease?
a) Lignocaine without adrenaline b) Prilocaine
c) Lignocaine with adrenaline 1:1000 d) Prilocaine and felypressin
Ans: A
51. Thirty seconds after the injection of 1.8 ml of local anesthetic containing 2%
lidocaine with 1:1,00,000 adrenaline patient loses consciousness. The most
probEaka dlagjitos’is is
a) Toxic reaction to the adrenaline b) Cerebral hypoxia
c) Tachycardia d) Toxic reaction to the lidocaine
Ans: B
52. Improper direction of the needle insertion during inferior alveolar nerve
block results in
a) Facial nerve paralysis b) Paraesthesia c) Hematoma d) Trismus
Ans: A
53. Increased incidence of reaction to L.A. will occur by
a) rapid rate of injection b) using an aspirating technique
c) addition of vaso constrictor to d) premedication with barbiturate the solution
Ans: A
54. Local anesthetic causing methemoglobinemia:
a) Prilocaine b) Etidocaine c) Lignoacine d) Bupivacaine
Ans: A

Page
21
55. The sensation of “tissue tearing” during inferior alveolar nerve block
indicates
a) passage through buccinator muscle
b) passage through an area of infection
c) barb on the needle d) large bore needle such as 19 no. needle
Ans: C
56. In a dental clinic after the administration of local anesthesia depression of
respiration occurred. It is a manifestation of
a) Trauma to a nerve trunk by the syringe needle
b) Toxic effects of the solution
c) Puncture of a blood vessel d) Use of an isotonic solution
Ans: B
57. Symptoms of epinephrine overdose following a local anesthetic injection
may include all of the following except:
a) restlessness b) hypotension c) apprehension d) palpitations
Ans: B
58. Which of the following is the cause of allergies to local anesthetics?
a) Slow elimination b) Improper technique
c) Slow detoxification d) Antigen and antibody reactions
Ans: B
59. Accidental intravenous injection of LA which contains a vasoconstrictor
may cause
a) convulsions and palpitations
b) unconsciousness and depressed respiration
c) increased respiration d) any of the above may occur
Ans: D
60. The maximum dosage of local anesthetic agent like lidocaine must be
reduced when used in combination with a CNS and/respiratory depressant
because, it may result in
a) seizures b) coma c) death d) All of the above
Ans: D
61. Hematoma formation is more frequent with
a) inferior alveolar nerve block b) posterior superior alveolar nerve block
c) greater palatine nerve block d) infraorbital nerve block
Ans: B
62. The most significant adverse consequence of accidental intravenous
administration of a local anesthetic is
a) bronchoconstriction b) hepatic damage
c) nerve damage d) seizures
Ans: D

Page
22
63. Jet injector primary use is
a) Topical anesthesia b) Supra-periosteal anesthesia
c) Pulpal anesthesia d) All of the above
Ans: A
64. Which of the following are theories of regional Analgesia?
a) Specificity theory and gate control theory
b) Specific receptor theory and gate control theory
c) Specific receptor theory and membrane expansion theory
d) Specificity theory and membrane expansion theory
Ans: C
65. Nerve membrane stabilization action of local anesthetic agents is due to
a) increasing conductance of potassium ions
b) creating negative after potentials
c) preventing conductance of Na ions from the exterior of the nerve to the
interior
d) preventing conductance of Na ions from the interior to the exterior of the
nerve
Ans: C
66. When compared to the non-myelinated nerve, myelinated nerve requires
a) higher concentration of anesthetic agent
b) lower concentration of anesthetic agent
c) same concentration of anesthetic agent
d) none of the above because no correlation
Ans: A
67. Myelinated nerve fibres have all of the following properties except:
a) conduction is slower in myelinated than in non-myelinated fibres
b) current discharges at nodes of ranvier
c) outer layer is of lipids
d) depolarisation occurs only at nodes of ranvier
Ans: A
68. Sensitivity to local anaesthetics is greater in:
a) type B fibres b) type C fibres
c) type A delta fibres d) fibres supplying the muscle spindles
Ans: B
69. The first sensation to be lost following administration of local anesthetic is
a) Proprioception b) Pain c) Touch d) Temperature
Ans: B
70. Of the following which tooth is most difficult to anesthetize by infiltration
technique alone,
a) maxillary canine b) maxillary central incisors
c) maxillary first molar d) maxillary first premolar Ans: C

Page
23
71. Dental extraction when indicated in a controlled haemophilic should be
done with
a) nerve block like inferior alveolar b) endotracheal general anaesthesia
c) field block d) periodontal ligament injection of L.A
Ans: D
72. Inferior alveolar nerve block is absolutely contraindicated in patients
suffering from one of the following disease
a) Thrombocytopenia b) Hemophilia
c) Hypoprothrombinemia d) Von willebrand’s disease
Ans: B
73. Nerve block which is given in case of severe trismus for achieving anesthesia
of inferior alveola nerve, long buccal nerve, lingual nerve, & masseteric nerve is
a) Gow-Gates technique b) Akinosis technique
c) V2 block d) Conventional inferior alveolar nerve block
Ans: B
74. Tuberosity technique is other name of which technique
a) Vazarani-Akinosis closed mouth technique b) Gow-Gates
c) IANB d) GP
Ans: A
75. In the extraoral technique for mandibular nerve block the needle after
contacting the pterygoid plate is directed
a) anteriorly b) posteriorly c) superiorly d) inferiorly
Ans: B
76. One wants to anesthetise the inferior alveolar nerve at the point at which it
exists from the mandibular canal, so the injection site most often located in
a) between first and second molars inferior to the root apices
b) between first and second molar superior to root apices
c) between first and second premolars inferior to root apices
d) between first and second premolars superior to root apices
Ans: C
77. The following anatomical land marks are useful in the closed block except
a) Occlusal plane of occluding teeth b) Anterior border of ramus
c) Mucogingival junction of the mandibular teeth
d) A & B
Ans: C
78. During the administration of posterior superior alveolar nerve block, the
landmarks that guid the operator are anterior border of coronoid process &
a) anterior border of the ramus b) mandibular 2nd premolar
c) pterygomandibular raphae d) palatine tonsils
Ans: A

Page
24
79. Among the following which nerve innervates the buccal gingiva of the
maxillary second premolar?
a) Anterior palatine nerve b) Buccal nerve
c) Middle superior alveolar nerve d) Posterior palatine nerve
Ans: C
80. The following nerves should be anesthetized for the removal of maxillary
first molar
1. Anterior palatine
2. Posterior palatine
3. Anterior superior alveolar
4. Middle superior alveolar
5. Posterior superior alveolar
a) 1, 3, 5 b) 1, 4, 5 c) 2, 3, 5 d) 2, 4, 5
Ans: C
81. The following areas are anesthetized by the anterior palatine nerve block
a) Part of the upper lip, lateral surface of nose and lower eyelid
b) Posterior portion of hard palate and overlying tissues upto the first premolar
c) Incisors canine and first premolar
d) Both hard and soft palates completely
Ans: B
82. In the extraoral infraorbital nerve block the following nerve is not
involved
a) Anterior superior alveolar b) Middle superior alveolar
c) Sphenopalatine d) Lateral nasal
Ans: C
83. Intraoral injection to block second division of the trigeminai nerve should
be given in the
a) foramen spinosum b) foramen ovale
c) pterygopalatine fossa d) infraorbital foramen
Ans: C
84. Following a maxillary tuberosity injection, within a few seconds, patient’s
face becomes extremely distended and swollen on the injected side.
Treatment is
a) Incision and drainage b) Antibiotic administration
c) Use of cold packs and pressure on d) Antihistamine administration the side
Ans: C
85. For the removal of root apex from the maxillary sinus, one should block
the following nerves
a) First division of trigeminal nerves b) Second division of trigeminal nerve
c) Third division of trigeminal nerve d) Infraorbital nerve
Ans: B

Page
25
86. During the administration of an inferior alveolar nerve block which of the
following muscles is usually penetrated with the needle
a) Buccinator b) Medial pterygoid
c) Lateral pterygoid d) Masseter
Ans: A
87. Properties of local anesthetics include all the following EXCEPT
a) Preferentially binds to sodium channels in open and inactivated state
b) Low affinity for channels in resting state
c) They are weak bases
d) Less effective in an environment with low extracellular pH
Ans: B
88. Usually hydrolysis of amide type of local anesthetic salts is facilitated by
a) tissue pH above 7.0 b) tissue pH between 5.0 and 7.0
c) tissue pH below 5.0
d) there is no relationship between pH and hydrolysis in amide type local
anaesthesia
Ans: A
89. Of the following in which condition local anaesthesia is ineffective?
a) Edema b) Localised infection c) Hematoma d) Anemia
Ans: B
90. Sodium bicarbonate when given with local anesthetics has which of the
following effect?
a) Increases rapidity of action
b) Helps intraneural spread of LA
c) Decreases onset and quality of block
d) Increases rate of drug absorption from site
Ans: A
91. The local anesthetic which can be used safely to patient who is allergic to
both lignocaine and procaime
a) Amethocaine b) Bupivacaine c) Mepivacaine d) Bucricaine
Ans: D
92. Amide type of local anesthetic agents undergo biotransformation primarily
in the
a) kidney b) liver
c) plasma d) excreted in unaltered form
Ans: B
93. Which of the following can be used as local anesthetic agent for infiltration
if a patient is allergic to amide and ester anesthetic derivatives
a) Nitrous oxide b) Bupivacaine
c) Phenylephrine d) Diphenhydramine
Ans: D

Page
26
94. The role of sodium bisulphate in a L.A solution is
a) Reducing agent b) Aesthetic agent
c) Vasoconstrictor d) Preservative
Ans: A
95. Frequently which of the following constitutes in amide type local anesthesia
causes allergic reaction
a) Lignocaine hydrochloride b) Methyl paraben
c) Adrenaline d) Contaminants in the solution
Ans: B
96. Which is not a useful function of adrenaline in local anesthetic
a) Decreased side effect of local anesthetia
b) Decreasing the duration of action of local anesthetic
c) Decreased anesthetic blood levels
d) Blood less field in surgery
Ans: B
97. The addition of hyaluronidase to a local anesthetic solution might
a) increase the duration of anesthesia b) limit the area of anesthesia
c) reduce bleeding d) enhance diffusion of local anesthetic
Ans: D
98. The effect of local anesthesia can be Increased by the addition of
a) adrenaline b) isoprenaline
c) dopamine d) felypressin [synthetic vasopressin]
Ans: A
99. Among the following, which condition is a containdictation to the use of
local anesthetic agent?
a) Parkinson’s disease b) Liver damage
c) Pregnancy (3rd trimester) d) Hypersensitivity to the drug
Ans: D
100. Local anesthetic cartridges contain small bubbles. These bubbles contain
a) oxygen gas b) carbon dioxide gas
c) nitrogen gas d) carbon monoxide gas
Ans: C
101. Lidocaine is used more commonly in dentistry because lidocaine
a) causes less depression of CNS
b) causes less cardiovascular collapse
c) causes lesser incidence of allergic reactions
d) is 50 times more potent than procaine
Ans: C
102. Which of following local anesthetics has intrinsic vasoconstrictive action?
a) Cocaine b) Procaine c) Xylocaine d) Bupivacaine
Ans: A

Page
27
103. Local anesthetia used in control of post operative pain is
a) Bupivacaine b) Prilocaine c) Lignocaine d) Mepivacaine
Ans: A
104. The maximum depth upto which topical anesthetics are effective is?
a) 1.0-2.0 mm b) 2.0-3.0 mm c) 3.0-4.0 mm d) 4.0-5.0 mm
Ans: B
105. In peripheral nerve block, which of the following local anesthetics has the
longest duration of action?
a) Bupivacaine b) Cocaine c) Lignocaine d) Prilocaine
Ans: A
106. Etidocaine and bupivacaine has duration of action hrs.
a) 2-4 b)5-7 c) 7-10 d) <2
Ans: A
107. Which of the following have the highest lipid solubility and protein
binding respectively?
a) Lidocaine and prilocaine b) Prilocaine and etidocaine
c) Etidocaine and articaine d) Procaine and articaine
Ans: C
108. Most commonly used local anesthesia
a) 1: 20,000 xylocaine Hcl b) 1: 50,000 xylocaine Hcl
C) 1: 10,000 xylocaine Hcl d) 1: 80,000 xylocaine Hcl
Ans: D
109. Toxic dose of lignocaine with adrenaline is
a) 5 mg/kg body weight b) 10 mg/kg body weight
c) 15 mg/kg body weight d) 20 mg/kg body weight
Ans: A
110. Each cartridge contain how many ml. of anesthetic solution
a) 0.9 ml b) 1.2 ml c) 1.8 ml d) 2.2 ml
Ans: C
111. The amount of vasoconstrictor in 1 ml of 2% Lignocaine solution with
1:2,00,000 adrenaline is
a) 0.5 mg b) 0.05 mg c) 0.005 mg d) 0.0005 mg
Ans: C
112. Maximum dose of xylocaine without adrenaline that can be given in 60kg
adults is
a) 500 mg b) 300 mg c) 400 mg d) 600 mg
Ans: B
113. The dose of epinephrine given in anaphlyaxis
a) 0.5 mg in 1: 1000 IM b) 0.5 mg in 1: 10,000 IM
c) 0.5 mg in 1: 500 IM d) 1 mg in 1: 100 IM
Ans: A

Page
28
114. Action of lignocaine is affected by all except
a) pH at the site of injection b) blood flow at the site of injection
c) vasoconstrictor in the L.A. solution
d) action of cholinesterase at the site of injection
Ans: D
115. Toxic effects seen after administration of a local anesthetic with
epinephrine are probably due to anesthetic agent if the following sign is present
a) Tachypnoea b) Tachycardia c) Drowsiness d) Sweating
Ans: C
116. Action of toxic doses of local anesthesia on CNS can be described as
a) first stimulating the CNS followed by depression
b) first depressing it followed by CNS stimulation
c) only depression of the CNS d) only stimulation of the CNS
Ans: A
117. The enzyme responsible for the maintenance of iron in hemoglobin in a
ferrous state and prevents the chances of methemoglobinaemia by converting
iron from ferric to ferrous state is
a) erythrocyte nucleotide diphorase b) erythrocyte nucleotide thiophorase
c) erythrocyte nucleotide reductase
d) erythrocyte nucleotide dehydrogenase
Ans: A
118. With overdosage of L.A. agent, one would observe
a) hypertension b) hypotension
c) no change in B.P. d) cardiac arrhythmias
Ans: B
119. The cause of trismus after inferior alveolar block anesthesia is
a) Damage to medial pterygoid muscle during injection
b) Myositis of the external pterygoid muscle
c) Submandibular cellulitis d) Stretching of the pterygomandibular raphe
Ans: A
120. Paraesthesia during a mandibular nerve block is due to
a) sodium conductance through the nerve
b) acid pH of the anesthetic solution
c) contact of the needle with the nerve trunk
d) poor technique
Ans: C
121. In systemic LA toxicity, there is
a) post depression convulsion b) post convulsion depression
c) convulsions d) depression
Ans: B

Page
29
122. Of the following which is treatment of sustained convulsive reaction to a
local anaesthesia
a) Oxygen & IV fluids b) Diazepam & oxygen
c) Phenytoin d) Slow acting barbiturates & oxygen
Ans: B
123. Toxicity of local anesthesia is reserved by
a) IV epinephrine b) IV nalorphine
c) IV barbiturates d) IV sodium bicarbonate
Ans: C
124. Cartridges should not be permitted to soak in alcohol because it
a) destroys vasoconstrictor b) is less effective
c) is warm in sensation
d) diffuses though rubber cap causing contamination
Ans: D
125. Local anesthetic agent acts by
a) Increasing rate of depolarization
b) Shortening rate of repolarization
c) Decrease the threshold potential
d) Increasing the threshold potential
Ans: D
126. Local anesthetic agent acts on
a) Nerve membrane b) Axoplasm
c) Epineurium d) Perineurium
Ans: A
127. The most acceptable theory which explain the action of local anesthesia is
a) Surface charge theory b) Calcium displacement theory
c) Membrane expansion theory d) Receptor binding theory
Ans: D
128. The ultimate action of binding the receptor by local anesthetic agent is
brought by its:
a) Hydrophilic component b) Lipophilic component
c) Intermediary chain d) R.N of amide agent
Ans: A
129. Which characteristic of L.A. agent is responsible for its penetration into the
nerve:
a) Lipid solubility b) Water solubility
c) Its ionization d) None of the above
Ans: A
130. Addition of vasoconstrictor to the local anesthetic agent causes:
a) Increase alkalinity of the solution b) Increase acidity of the solution
c) Has no effect on the L.A. d) None of the above Ans: B

Page
30
131. Sodium bisulfate used in dental L.A. cartridge acts as:
a) Antioxidant for adrenaline b) Antioxidant for lignocaine
c) Antibacterial for lingocaine d) Not used any more
Ans: A
132. Sodium bisulfates has the following effect on the L.A. Solution:
a) Slows down its onset of action b) Deceases its duration
c) Increases its Ph d) Has no effect
Ans: A
133. Increase concentration of local anesthetic agent from 2% to 4% would
have:
a) Rapid onset & prolonged duration
b) Onset would not be affected but duration would be prolonged
c) No change in action
d) Rapid onset but duration would not be affected
Ans: D
134. Tachyphylaxis (tolerance) occurs due to:
a) Increase dose of L.A. b) Increase dose of vasoconstrictor
c) Repeated use of L.A. d) Allergy to sodium bisulfate
Ans: C
135. Which of the following belongs to ester group of local anesthesia:
a) Bupivacaine b) Benzocaine
c) Etidocaine d) Mepivecaine
Ans: B
136. Which of the following is not an ester L.A. :
a) Propoxcaine b) Procaine
c) Prilocaine d) Pipacaine
Ans: C
137. All L.A. crosses the placenta:
a) Except mepivacaine b) Statement is true
c) Statement is false d) Except lignocaine
Ans: B
138. Which of the following local anesthetics cross the blood-brain barrier:
a) Lingocaine b) Etidocaine
c) Bupivacaine d) All of the above
Ans: D
139. Ester type of L.A. are metabolized in:
a) Liver b) Plasma c) Kidney d) Lungs
Ans: B

Page
31
140. A patient had history of apnea during administration of muscle relaxant
(sccinyle choline) should not be given:
a) Ester type L.A. b) Amide type L.A.
c) Vasoconstrictor d) Sodium bisulfate
Ans: A
141. A patient complains a history of hepatitis one month ago should be
preferably given which local anesthetic agent:
a) Lingocaine b) Procaine
c) Procainamide d) Bupivacaine
Ans: B
142. L.A. mainly excreted via:
a) Lung b) Kidney
c) Uterus d) Fecal route
Ans: B
143. When vasoconstrictor is contraindicated the ideal L.A. to be used:
a) Lingocaine b) Mepicaine
c) Bupivacaine d) Cocaine
Ans: B
144. Which of the following is least toxic L.A. :
a) Lingocaine b) Mepicaine
c) Propoxycaine d) Bupivacaine
Ans: D
145. Which of the following is a long acting L.A. :
a) Mepivacaine b) Bupivacaine
c) Prilocaine d) Propoxycaine
Ans: B
146. The allergic reactions commonly seen following use of L.A. is due to:
a) Lingocaine b) Vasoconstrictor
c) Methyl parapen d) Sodium bisulfate
Ans: C
147. Glass L.A. cartridge should be sterilized by:
a) Autoclaving b) Dry heat
c) Cold sterilization d) Non of the above
Ans: D

Page
32
EXODONTIA
1. Most important principle during extraction
a) Least trauma to bone while extracting whole tooth out
b) Least trauma to mucosa while extracting the whole tooth out
c) Least trauma to both bone and mucosa while extracting the tooth in pieces
d) None of the above
Ans: C
2. A primary rotatory movement is used in extraction of
a) mandibular central and lateral incisors
b) maxillary central incisor and mandibular II premolar
c) maxillary I premolar d) maxillary and mandibular canines
Ans: B
3. Appropriate time to administer analgesic to reduce post operative pain
associated with removal of two mandibular molars
a) When pain becomes severe b) Before the effect of LA wears off
c) The morning of surgery d) Only after the return of sensation
Ans: B
4. When forceps are to be utilized for removal of a tooth, the first direction for
the force to be applied is
a) occlusally b) buccally c) lingually d) apically
Ans: D
5. In multiple extractions, the order of extraction usually
1) Maxillary teeth before mandibular
2) Mandibular teeth before maxillary
3) Posterior teeth before anterior
4) Anterior teeth before posterior
a) 1 & 3 b) 2 &4 c) 2 & 3 d) 1 &4
Ans: A
6. “Odontectomy” is synonymous to
a) Transalveolar extraction b) Intra alveolar extraction
c) Extraction under local anesthesia d) Extraction under general anesthesia
Ans: A
7. “Line of withdrawal” of a tooth is mainly determined by
a) the size of the crown b) the root pattern
c) shape of the crown d) size of the roots
Ans: B

Page
33
8. The following design parameters should be followed to prepare a
mucoperiosteal flap except
a) Flap should provide adequate visibility & accessibility
b) Free margin broader than base
c) Ragged margins should be avoided
d) Flap should be a full thickness mucoperiosteal flap
Ans: B
9. Limited accessibility is one of the most disadvantages of the following flap
a) Envelop flap b) Semilunar flap
c) Three cornered flap d) Four cornered flap
Ans: B
10. Recontouring of the residual ridge in the early part of extraction of period
occurs primarily by resorption of the
a) buccal-lingual cortical bone b) labial – buccai cortical bone
c) lingual cortical bone d) crestal bone
Ans: B
11. A 20 years old patient requires extraction of all erupted teeth because of
extensive decay. He has two maxillary third molar impactions. X-ray film shows
bony impactions. The molars should be
a) extracted along with other teeth.
b) retained because no changes of eruption.
c) retained under a denture until they erupt.
d) exposed surgically & can be used for prosthetic retention.
Ans: C
12. Extraction of an isolated residual maxillary molar offers the technical hazard
of
a) the development of maxillary sinusitis
b) displacement into the infratemporal fossa
c) fracture of the tuberosity (or) floor of the antrum
d) producing a nasal oral fistula
Ans: C
13. The maxillary first molar is extracted by forceps method. The healing of the
socket can be described as
a) Healing by primary intention b) Granulomatosis
c) Healing by secondary intention d) Epithelialization
Ans: C

Page
34
14. The immediate step following the forceps extraction of a maxillary central
incisor with a periapical radiolucency is
a) Examine the root apex for soft tissues pathology
b) Pack the socket with a medicament
c) Currette the alveolus, if tissue was not removed with extraction
d) A & C
Ans: D
15. Which of the following impactions of mandibular molars are easy to
extract?
a) Mesioangular b) Distoangular
c) Vertical d) Horizontal
Ans: A
16. According to Pell & Gregory classification, if occlusal plane of impacted
tooth is between occlusal plane and cervical line of 2nd molar, type of
impaction is of
a) Class A b) Class B c) Class C d) Class D
Ans: B
17. Apical notch in radiograph is commonly is seen in the following impactions
a) Mesioangular and distoangular impaction
b) Vertical and horizontal impaction
c) Mesioangular and horizontal impaction
d) Horizontal and distoangular impaction
Ans: A
18. An approach to an impacted mandibular molar is achieved by
a) envelop flap b) bayonet flap
c) V shaped flap d) All of the above
Ans: D
19. The Nerve most likely to get injured during incision placement in lower
wisdom tooth surgery is
a) lingual nerve b) Inferior dental verve
c) Mylohyoid nerve d) Mental nerve
Ans: A
20. During extraction of lower impacted right molar, bone is removed
a) to expose maximum dimension of tooth b) upto CE junction
c) upto furcation area d) upto half of roots
Ans: A
21. Extraction of disto-angular impaction of mandibular 3rd molar can cause
a) slippage in lingual pouch b) fracture of ramus of mandible
c) excessive haemorrhage d) dry socket
Ans: B

Page
35
22. Contributing factor to pericoronitis of impacted mandibular third molar is
a) trauma by opposing tooth b) previous radiation therapy
c) systemic disease d) infected follicular cyst
Ans: A
23. After surgery of 3rd molar, patches of anesthesia on chin is due to
damage of
a) lingual nerve b) inferior alveolar nerve
c) mental nerve impaction d) None of the above
Ans: B
24. Which maxillary impactions are most likely to be displaced into the antrum
or the infratemporal fossa with incorrect technique during the extraction
a) Distoangular b) Mesioangular
c) Vertical d) Faciolingual
Ans: A
25. A palatal flap is required for removal of a maxillary impacted canine near
the midline. Palatal flap should be
a) started with a vertical incision in the midline
b) reflected from the neck of the teeth
c) made with a semilunar incision
d) None of the above palatal flap should be avoided
Ans: B
26. Ten teeth have been removed for a patient who was premedicated. The
proper position of the patient in the recovery room is
a) Head elevation 30° with patient on his back
b) Trendelenberg position, patient head is almost parallel to the floor
c) Reverse trendelenburg position d) Supine position
Ans: A
27. While extracting a multirooted tooth, the crown fractures, the prudent next
step is to
a) divide the roots with a bur b) reflect a flap
c) reapply the forceps in a more
d) leave the roots in place and observe apical direction
Ans: A
28. During the dental procedure suddenly patient became “light headed,
diaphoretic” and then became unconscious. The diagnosis is
a) Shock b) Syncope c) CVA d) Hyperventilation
Ans: B
29. In a vasovagal syncope, which of the following does not occur?
a) Hypotension and tachycardia b) Constriction of pupil
c) Vomiting d) Cold extremities
Ans: B

Page
36
30. Most common cause of post extraction bleeding is
a) bleeding disorders
b) failure of the patient to follow post extraction instructions
c) due to the analgesics, such as aspirin
d) patient with the anticoagulant therapy
Ans: B
31. Reactionary hemorrhage occurs after extraction because of
a) broken roots b) high blood pressure
c) sharp interdental septum d) gingival laceration
Ans: B
32. Hemorrhage after dental extraction procedure can be prevented by
a) applying pressure b) ligating bleeding major vessels
c) properly designing and carefully reflecting mucoperiosteal flaps
d) All of the above
Ans: D
33. Among the following, which is best hemostatic agent for control of local
hemorrhage?
a) Gel foam b) Surgicel c) Bone wax d) Vit. K
Ans: B
34. If there is visible bleeding from an isolated major vessel, it is best controlled
by
a) systemic administration of Vit K b) electro coagulation
c) clamping and ligation d) B & C
Ans: D
35. Ecchymosis and hematoma are treated with
a) intermittent ice pack b) continous ice pack
c) intermittent hot pack d) pressure & pack
Ans: A
36. A swelling overlies an extraction wound and it crackles on palpation. Most
likely diagnosis is
a) Ecchymosis b) Cellulitis
c) Emphysema d) Empyema
Ans: C
37. All of the following statements about inhaled tooth fragment are true
EXCEPT
a) It usually enters the right bronchus. b) It may cause lung abscess.
c) It may cause bronchiectasis.
d) It will be coughed out spontaneously, no treatment is necessary.
Ans: D

Page
37
38. During extraction of maxillary third molar, the maxillary tuberosity is
fractured and remains attached to periosteoum with an intact blood supply.
The treatment of choice to
a) refer the case to an oral surgeon b) remove the fractured tuberosity
c) reposition and stablize loss of the tuberosity
d) send the patient to preventive dentistry with instructions to prevent bleeding
Ans: C
39. Displacement of a root into the maxillary sinus during extraction is most
likely to happen during the extraction of the maxillary
a) canine b) deciduous first molar
c) first molar d) second molar
Ans: C
40. After the extraction of maxillary first molar, a communication is found
between the palatal socket and a disease free maxillary sinus which measure 0.2
cm. The best treatment is
a) Allow the clot to form, advise proper home care
b) Primary closure and antihistamine
c) Gold foil closure d) Caldwell Luc operation
Ans: A
41. During the extraction of maxillary first molar, the palatal root tip of molar
slipped into the maxillary sinus. Proper way to approach to recovery is through
the
a) same socket by enlarging the opening through which the root entered the
sinus
b) maxillary incisive fossa
c) maxillary incisive fossa medial to canine
d) lateral nasal wall in the middle meatus of the nose
Ans: B
42. A 23 years old female patient complains of decreased mouth opening since
4 days. This could be due to
a) impacted third molar b) oral submucous firbrosis
c) oropharyngeal fibrosis d) bony ankylosis of TMJ
Ans: A
43. Five days after extraction of an impacted mandibular third molar, your
patient presents with hard, painful swelling of 12 hours' duration near angle of
the mandible. This is most likely due to
a) Angioneurotic edema b) Hematoma
c) Infection d) Surgical trauma
Ans: C
44. Severe pain in dry socket is due to release of
a) kinins b) plasmins c) infection d) lysis of fibrin Ans: A

Page
38
45. Treatment of choice for a localised osteitis or dry socket following
extraction of a tooth is
a) Topical antibiotic application in the socket
b) Systemic antibiotic administration
c) Debridement of the wound by irrigation and placement of sedative
d) Curetting the bony walls of the socket to induce bleeding and clot formation
Ans: C
46. A patient returns several days after extraction of three adjacent mandibular
molars and complains of pain similar to dry socket. The first step is to
a) Curette the socket area thoroughly in an attempt to establish new blood clot
b) Explore the sockets gently with curette
c) Irrigate the socket and place the dressing
d) Radiographic examination of the area
Ans: B
47. Of the following which teeth need primarily rotatory movement to extract
them
a) Maxillary first and second premolar
b) Mandibular canine and second premolar
c) Mandibular central and lateral incisor
d) Mandibular first and second premolars
Ans: B
48. The extraction of which of the following is most difficult in maxillary arch
a) Lateral incisor b) First premolar
c) Second premolar d) Canine
Ans: B
49. Extraction of a tooth during acute infection
a) can cause extensive spread of infection
b) helps drainage and relieves pain if proper antibiotic is given and its adequate
blood level is reached
c) can cause sudden death due to pulmonary embolism
d) can lead to trigeminal neuralgia in post operative period
Ans: B
50. The beak of an extraction forceps is designed such a way that the extraction
pressure is transmitted to the.
a) crown of the tooth b) alveolar bone
c) root of the tooth d) cementoenamel junction of the tooth
Ans: C
51. Flap with no vertical incision is called:
a) trapezoidal b) triangular c) envelope d) semilunar
Ans: C

Page
39
52. Which one of the following is not an indication for open view extraction
a) Hypercementosis b) Bone sclerosis
c) Abnormal root curvature d) Carious teeth with intact crown
Ans: D
53. A tooth is resistent to luxation with forceps. Its removal is best achieved by
a) fracture the crown with forceps
b) fracture the crown with chisel and roots with forceps
c) transalveolar method d) split tooth technique
Ans: C
54. Among the following, the factor of least importance in alveolar flap
design is
a) linear incision b) closure over the bone
c) tension free closure of the flaps d) base relationship to length
Ans: A
55. The following are indication for suturing following exodontia except
a) blood clot protection b) reapproximation of mucoperiosteal flaps
c) immobilisation of tissues d) hemostasis
Ans: A
56. When mandibular teeth are to be extracted, the patient should be
positioned in such a way that the occlusal plane of the mandibular arch is
a) parallel to the floor when the teeth are closed together
b) parallel to the floor when the mouth is wide open
c) perpendicular to the floor when the teeth are closed together
d) perpendicular to the floor when the mouth is wide open
Ans: B
57. The left hand of the operator should serve the following purpose when
extracting a mandibular tooth
a) Support the mandible b) Protect other teeth
c) Retract soft tissues d) All of the above
Ans: D
58. A mandibular left second molar is unerupted in a 14-year-old boy.
Radiological examination reveals a dentigerous cyst surrounding the unerupted
tooth. The treatment of choice is
a) Observe for at least 2 years
b) Aspirate and send the contents of the cyst to biochemical analysis
c) Uncover the crown and keep it exposed
d) Extract the tooth under local anesthesia
Ans: C

Page
40
59. Prophylactic removal of unerupted teeth is accomplished with best results
at certain stage of tooth development. The best time to remove such a tooth is
a) when two thirds of the root are formed
b) when the root is completely formed
c) before any of the root is formed
d) when the crown of the tooth is above the cementoenamel junction of
adjacent tooth
Ans: A
60. The healing process that takes place following approximation of the flaps
are described as
a) Healing by first intention b) Healing by second intention
c) Healing by granulomatosis
d) Hematoma formation followed by organization
Ans: A
61. Of the following most difficult impaction to extract is
a) Mesioangular b) Vertical
c) Distoangular d) Horizontal
Ans: C
62. Impacted mandibular third molar classification is usually based on
a) relation of the tooth to the ramus of the mandible
b) the position of the tooth in relation to the long axis of the second molar
c) the relative depth of the third molar in bone d) All the above
Ans: D
63. The “red line” in Winter’s classification of impacted teeth represents
a) The depth at which the impacted teeth are located
b) The angulation of the 2nd molar
c) The summit of the alveolar bone covering the impacted tooth
d) Relation of the 3rd molar to the ramus
Ans: A
64. X ray for third molar reveals that inferior nerve canal becomes thinner
along the roots of third molar. The most possible cause is
a) Root perforated by nerve b) Nerve goes between root of teeth
c) When canal crosses the root apex d) Nerve encircles the root
Ans: C
65. In the extraction of mandibular third molars, the main reason why the
posterior incision should be placed more buccally is
a) to prevent damage to lingual nerve
b) incision should be on the sound bone
c) to prevent damage to retromolar artery d) All of the above
Ans: D

Page
41
66. Which is not injured in lower third molar extraction?
a) Inferior alveolar nerve b) Inferior alveolar artery
c) Lingual nerve d) Lingual artery
Ans: D
67. The advantage of lingual split technique for extraction of mandibular third
molar impacted teeth is /are
a) Bone loss is minimal b) Easy and quick method
c) Tissue trauma is minimal d) All of the above
Ans: D
68. During the extraction of mandibular third molar it is noted that the distal
root is missing the root tip is most probabiy in the
a) submental space b) submandibular space
c) pterygomandibular space d) paraphyaryngeal space
Ans: B
69. Best treatment for pericoronitis associated with impacted mandibular third
molar is
a) irrigating under the operculum b) antibiotic and analgesic therapy
c) extraction of impacted third molar d) operculectomy
Ans: C
70. Commonest complication after removal of mandibular 3rd molar
a) Lingual nerve damage b) Dry socket
c) Fracture of mandible d) Bleeding
Ans: B
71. Impacted canines
a) Most likely to be ankylosed.
b) They may damage roots of other teeth.
c) Good function is impossible without them.
d) They will not erupt until root formation is complete.
Ans: B
72. During reflecting a palatal mucoperiosteal flap for removal of an impacted
canine structure passing through the incisive canal is averted. The sequelae to
this is
a) of no clinical significance
b) loss of a triangular mucosal flap behind the incisors and canine due to loss of
blood supply
c) resorption of bone around the canal d) anesthesia of the anterior palate
Ans: A

Page
42
73. Post operative edema can be minimized by
a) careful surgical manipulation of hard and soft tissues.
b) administration of antihistamines pre-operatively.
c) application of heat immediately after the procedure.
d) application of heat and cold simultaneously.
Ans: A
74. In a patient with syncope the pupils are?
a) constricted b) dilated-fixed and non-reacting to light
c) constricted and non-reacting to light d) dilated and fixed
Ans: D
75. Which of the following is the principle action of ammonia in syncope
a) Respiratory stimulant b) Vagal stimulant
c) Vasomotor stimulant d) Inhibitor of vasomotor tone
Ans: A
76. Instead of responding to treatment for syncope, the patient’s pulse and
respiration become weak and irregular with attendant cyanosis. The first
resuscitative measure should be to
a) support circulation by injecting 1:1000 adrenaline.
b) begin closed chest cardiac massage.
c) support respiration with oxygen through an open airway.
d) place a paper bag over patient’s face and raise blood CO2.
Ans: C
77. Following may cause post operative hemorrhage
a) Liver disease b) Aspirin &. other salicylates medication
c) Prolonged use of broad d) All of the above
Ans: D
78. Which of the following will cause post operative haemorrhage?
a) Salicylates b) Liver cirrhosis
c) Prolonged antibiotic administration d) All of the above
Ans: D
79. All of the following are various methods to control intraoperative
hemorrhage except
a) Gauze sponge pressure b) Artery application to the open vessels
c) Infiltration with lidocaine
d) Bone compression for surface bleeders Infiltration with lidocaine can not be
used.
Ans: C

Page
43
80. A patient complains of persistent bleeding five hours after the extraction.
Initial treatment is to
a) Advise bleeding time and clotting time and other laboratory investigations
b) inject a local anesthetic solution into the area to provide vasoconstriction
c) remove clots and examine the area to locate the source of bleeding
d) have the patient bite on a gauge socked with adrenaline
Ans: C
81. Four days after multiple dental extractions, an elderly fair skinned woman
complains of black and blue marks over her neck.
The most probable diagnosis is
a) Polycythemia b) Leukemia
c) Post operative ecchymosis d) Manifestation of basal cell carcinoma
Ans: C
82. Three days after extraction of teeth for an immediate denture, patient
complains of a diffuse, non-painful, yellow submandibular and suprasternal
discoloration of the skin. The dentist should
a) Advise antibiotics b) Apply heat and advise analgesics
c) Apply cold compressions d) None of the above
Ans: D
83. One day after the extraction, patient came with the complaint of a
rubbery, non-tender midfacial swelling. The most likely diagnosis
a) Abscess b) Fibroma
c) Post operative ecchymosis d) Inflammatory edema
Ans: C
84. Most common complication arising out of maxillary third molar extraction
a) Formation of oroantral fistula b) Opening in nasal floor
c) Fracture of maxillary tuberosity d) Tooth slipped in pharyngeal spaces
Ans: C
85. When a root is accidentally displaced into the maxillary antrum, the dentist
should
a) enlarge the socket to gain access b) place a drain into the socket
c) refer to an experienced surgeon for root removal
d) pack the socket with Gelfoam to prevent fistula formation
Ans: C
86. A mandibular second moral (mandibular third molar) is extracted. It is
found that the distal root is fractured and the tip is missing. When the dentist
attempts to recover it, and finds that his instrument will pass downward and
into the lingual soft tissue, the tip is most likely to be in the
a) submental space b) sublingual space above the mylohyoid muscle
c) submandibular space below the mylohyoid muscle
d) parapharyngeal space Ans: C

Page
44
87. Trismus after extraction most likely results from
a) passing the needle through medial pterygoid muscle while injecting
b) injecting the solution near a branch of facial nerve
c) injecting the solution near a branch of trigeminal nerve
d) systemic administration of LA
Ans: A
88. Three days after the extraction of maxillary first molar patient developed a
fluctuant palatal abscess. The treatment of choice is
a) Caldwell Luc procedure b) Heat and cold therapy
c) I & D and antibiotic therapy
d) Only antibiotic therapy after culture and sensitivity tests
Ans: C
89. Few days after a dental extraction, patient came with marked pyrexia and
complaining of impairment of labial sensation and marked tenderness on extra
oral palpation. Most probable diagnosis is
a) Dry socket b) Acute osteomyelitis
c) Post operative haematoma d) Nerve damage
Ans: D
90. A mandibuiar molar was extracted because of advanced caries. After one
month patient developed an extra oral indurated swelling near the previous
extraction site. Multiple draining sinuses appeared and drained pus with
yellow granules. The most probable diagnosis is
a) Histoplasmosis b) Tuberculosis
c) Actinomycosis d) Streptococcal infection
Ans: C
91. Patient complains of pain and foul smell on third day after extraction.
Treatment is
a) Irrigate the socket and place sedative dressing
b) Curettage, sedative dressing
c) Search for the roots d) None of the above
Ans: A
92. Diagnosis of dry socket is done by
a) history b) clinical examination
c) radiographs d) None of the above
Ans: A
93. Management of dry socket (localised alveolar osteitis) include all of the
following except
a) Curetting the socket and promote bleeding
b) Normal routine irrigation to flush out the debris
c) Placing a sedative dressing to protect the exposed bone
d) Advising analgesics as an adjunctive treatment Ans: A

Page
45
FACIAL SPACE INFECTIONS

1. The greatest barrier to infection is


a) conncetive tissue b) epithelium
c) muscle d) fascia
Ans: D
2. Odontogenic infections are mostly caused by
a) MIXED bacteria b) Anaerobic bacteria
c) Aerobic bacteria d) Streptococci
Ans: A
3. Among the following, which results in sudden swelling in the area
of a dental
a) Drug incompatibility b) Paravascular injection
c) Infection d) Vascular penetration
Ans: D
4. After incision and drainage of an abscess, the infectious process has failed to
regress inspite of the patient being on high doses of an antibiotic, it would be
wise to
a) repeat culture and sensitivity tests b) insert a large drain
c) augmenting antibiotic action by administration of parenteral proteolytic
enzymes
d) debride and irrigate the area with a fibrinolylic agent
Ans: A
5. Incision & Drainage (I & D) in an area of acute infection should be
performed when
a) induration has occurred b) localization has occurred
c) acute pain is present d) There is fever which is above 102°F
Ans: B
6. A 9-year-old child with carious maxillary lateral incisor, got small swelling
over the respected area. In 48 hours, the swelling enlarged and reached to the
lower border of eye, very sensitive, hot and, painful to the touch stimuli. The
lymph node is palpable, fever present and the swelling rebounds on pressure.
The recommended treatment is
a) Antibiotic and hot fomentat b) Incision and drainage
c) Injection of sclerosing unit d) Application of pressure bandage
Ans: B
7. Pericoronitis is seen in relation to
a) impacted third molars only b) around incompletely erupted crown
c) completely erupted crowns only d) None of the above
Ans: B

Page
46
8. Antibiotic medication for pericoronitis should be advised
a) If extraction is delayed (or) postponed b) Before surgery
c) Routinely to treat pericoronitis d) If trismus and fever present
Ans: D
9. Which of the following is not a primary mandibular space?
a) Buccal b) Sublingual
c) Submandibular d) Pterygomandibular
Ans: D
10. Ludwig’s angina involves…………. spaces
a) submandibular b) submental
c) sublingual d) All
Ans: D
11. Ludwigs angina is usually caused by
a) streptococci and various mixed anerobes
b) anaerobic infection by prevotella and fusobacterium
c) paramyxo virus d) Candida species
Ans: A
12. Which of the following is not associated with the posterior aspect of lateral
pharyngeal space?
a) Vagus nerve b) Lymph node
c) Glossopharyngeal nerve d) Carotid sheath
Ans: C
13. Which of the following is a secondary site for spread of odontogenic
infection involving pterygo mandibuiar space?
a) Infra temporal space b) Canine space
c) Buccal space d) Sublingual space
Ans: A
14. In pterygomandibuiar space the inferior alveolar nerve passes
a) medial to stylomandibular ligament
b) anterior to the deep tendon of the temporal muscle
c) lateral to the sphenomandibular ligament
d) superior to lateral pterygoid muscle
Ans: C
15. The infection from a lower third molar pericoronal area spreads mostly to
a) submandibular space b) submental space
c) pterygomandibular space d) buccal space
Ans: C
16. The distinguishing feature of masticatory space infection is
a) pain b) dysphagia c) trismus d) swelling
Ans: C

Page
47
17. Retropharyngeal space infection is mainly due to spread of
a) cervical tuberculosis b) meningo encephalitis
c) mumps d) odontogenic infections
Ans: D
18. The following space infection may cause severe respiratory difficulty and
require tracheostomy
a) Pterygopalatine space b) Canine space
c) Temporal space d) Parapharyngeal space
Ans: D
19. The facial space that is divided by the styloid process into anterior and
posterior compartments is
a) Pterygomandibular b) Lateral pharyngeal
c) Retropharyngeal d) Infratemporal
Ans: B
20. A mandibular dental infection from the mandibular first molar which exists
the buccal cortical plate above the muscle attachment will cause abscess of
a) buccal space b) masseter space
c) vestibular sulcus d) masticatory space
Ans: C
21. Subperiosteal abscess, penetrating deep is seen after extraction of
a) maxillary 3rd molar b) mandibular 3rd molar
c) maxillary 1st molar d) mandibular 1st molar
Ans: B
22. Palatal abscess most commonly results from infection of
a) maxillary centrals b) maxillary laterals
c) maxillary canine d) maxillary premolars
Ans: B
23. Infection from the dangerous area of the face spreads to the cavernous
sinus via
a) neural sheath b) emissary veins
c) orbital fissure d) ethmoidal air sinuses
Ans: B
24. The incision for drainage in Ludwig’s angina extends
a) up to the neck b) to the angle of the mandible
c) floor of mouth d) All the above
Ans: C
25. Osteomyelitis
a) never occurs in infants
b) in acute cases fracture mandible is very common
c) of mandible, can show symptoms of lip Paresthesia
d) produces no lymphadenopathy Ans: C

Page
48
26. Involucrum is
a) dead bone b) new live bone
c) previous live bone d) sclerotic bone
Ans: B
27. Osteomyelitis of the jaw can be cured by
a) resection b) physiotherapy
c) sequestrectomy with antibiotic d) drainage treatment
Ans: C
28. To drain pus from an abscess, the surgeon should
a) cut an eclipse from the abscess surface to allow for a drain
b) aspirate the contents
c) penetrate into abscess cavity and probe with an artery to allow for flow of
pus
d) cut only the mucosa and skin
Ans: C
29. The technique of doing the incison and drainage of an abscess is known as
a) Andrew’s method b) Hilton’s method
c) Nilaton’s method d) Killey’s method
Ans: B
30. Hilton’s method deals with
a) decompression of a cyst b) drainage of an abscess
c) biopsy removal d) preanesthetic check-up
Ans: B
31. In a 19-year-old patient with a swelling over the left angle of the mandible,
temperature of 38°c and negative history of trauma, one should suspect
a) Spontaneous fracture of the mandible b) Pericoronal infection
c) Mumps d) Sjogren’s syndrome
Ans: B
32. If one treats an abscess with antibiotics without I & D it may cause
a) non-suppurative inflammatory reaction b) formation of antibioma
c) healing with scar formation d) None of the above
Ans: B
33. Buccal space is
a) Primary maxillary space b) Primary mandibular space
c) Both A and B d) Secondary fascial space
Ans: C
34. The following are the characteristic features of Ludwig’s angina
a) Raised tongue b) Bilateral submandibular swellings
c) Dysphagia d) All of the above
Ans: D

Page
49
35. Which of the following is not true of Ludwig’s angina?
a) Involves bilateral submandibular, sublingual, submental space.
b) Characterized by cellulitis, brawny oedema and lymphadenopathy.
c) Difficulity in swallowing and breathing
d) Characterized by deviation to unaffected side
Ans: D
36. A patient complaining of trismus and difficulty in swallowing, extraoral
clinically no swelling with an anterior bulging of half the soft palate and the
anterior tonsillar pillar with deviation of uvula to the unaffected side is suffering
from
a) Sub massetrric space infection b) Parapharyngeal space infection
c) Pterygomandibular space infection d) Peritonsillar infection
Ans: C
37. The roof of pterygo–mandibular space is formed by:
a) Temporalis muscle b) Medial pterygoid muscle
c) Cranial base d) Lateral pterygoid
Ans: D
38. Which of the following is not present in pterygomandibular space?
a) Auriculo temporal nerve b) Lingual nerve
c) Mandibular nerve d) Mylohyoid nerve
Ans: A
39. Masticator space (Mainly pterygomandibular) communicates posteriorly
with the following spaces
a) Masseteric and sub-lingual space
b) Parotid and lateral pharyngeal space
c) Sub-mandibular and sub-lingual space
d) Lateral pharyngeal and pterygo manibular space
Ans: B
40. In pericoronal abscess related to distoangular impacted lower third molars
the infection may spread to
a) submasseteric space b) sublingual space
c) submental space d) buccal space
Ans: A
41. The most dangerous type of spread of infection from apical abscess is to
a) infra temporal fossa b) pterygoid
c) parapharyngeal space d) submandibular space
Ans: C
42. Lateral pharyngeal space is not connected to
a) buccal space b) sublingual space
c) submandibular space d) retropharyngeal space
Ans: A

Page
50
43. Trismus associated with infection of lateral pharyngeal space is related to
irritation of the
a) buccinators b) masseter
c) lateral pteryrgoid d) medial pterygoid
Ans: D
44. Infection from periapical region of molar penetrates below the buccinator
burrows suh-periostealfy into
a) sub-masseteric space b) pterygomandibular space
c) sub-lingual space d) parotid space
Ans: A
45. A periapical abscess of a mandibular second molar space spreads most
commonly to the
a) submandiluar space b) temporal space
c) sublingual space d) infra temporal space
Ans: A
46. Infection from maxillary first molar drains into
a) submandibular space b) infratemporal space
c) buccal space d) infraorbital space
Ans: C
47. After extraction of upper central incisor, patient develops ophthalmoplegia,
meningitis and lateral rectus paralysis. The diagnosis is
a) Cavernous sinus thrombosis b) Not related
c) Cellulitis d) Ludwig’s angina
Ans: A
48. Abducent nerve paralysis is the earliest finding in the case of
a) lefort III fracture b) nasoorbito ethmoid fracture
c) zygomatic fracture d) cavernous sinus thrombosis
Ans: D
49. Cavernous sinus thrombosis following infection of maxillary and anterior
teeth most often results from spread of infected emboli along the
a) pterygoid plexus b) ophthalmic vein
c) facial artery d) angular artery
Ans: B
50. The severe complication of the canine space infection is
a) erosion of internal carotid artery b) erosion of external carotid artery
c) cavernous sinus thrombosis d) respiratory paralysis
Ans: C
51. Death in Ludwig’s angina occurs due to
a) sepsis b) respiratory obstruction
c) cavernous sinus thrombosis d) carotid blow-out
Ans: B

Page
51
52. Which of the following features is not associated with acute osteomyelitis
of mandible
a) Severe pain b) Purulent exudate
c) Paresthesia of lower lip d) Radiographic evidence of bone destruction
Ans: D
53. Treatment of Garre’s osteomyelitis is
a) incision and drainage b) sequestrectomy
c) saucerization. d)surgical recontouring
Ans: D
54. Hyperbaric oxygen is indicated for
a) obstructive lung diseases b) osteoradionecrosis
c) cardiac failure d) renal diseases
Ans: B
55. Osteomyelitis of the TMJ can be treated by
a) condylectomy
b) irradiation
c) incision and drainage, if needed condylectomy
d) None of the above
Ans: C

Page
52
BIOPSY PROCEDURES

1. 60-year-old woman with a past history of adenocarcinoma of the breast


complaining of an ulcer on the right lateral border of her tongue. Fractured
cusp on the mandibular right molar which exposes a sharp margin of denta!
amaigum is found during routine clinical examination. Treatment of choice is
a) Immediate biopsy of the lesion
b) Restoration of the tooth and biopsy in 2 weeks if necessary
c) Restoration of the tooth
d) Restoration of the tooth with biopsy in 2 months, if the lesion does not heal
Ans: B
2. Of the following in which condition a biopsy should not be performed on a
tissue. When the tissue
a) responds to local treatment b) is associated with paresthesia
c) suddenly enlarges d) B & C
Ans: A
3. A man has I x 1.5 cm pedunculated lesion on the soft palate which has a
rough, “warty” surface but is the same colour as adjacent mucosa. Appropriate
management of this lesion is to:
a) Perform an incisional biopsy b) Perform excisional biopsy
c) Scrape for exfoliative cytology d) Observe for two weeks
Ans: B
4. Of the following conditions, where is an incisional biopsy indicated
a) 4 cm hemangioma of the tongue
b) 3 cm leucoplakia of the soft palate
c) 0.5 cm papillary fibroma of the
d) 1.5 cm exostosis of jaw gingival
Ans: B
5. Which one of the following should not be done during in incisional biopsy
of soft tissue
a) Place suture through the intended specimens before removing it
b) Infiltrate local anesthetic solution around the intended site
c) Place the specimen in saline if 10% formalin is unavailable
d) Obtain Edge of the lesion along with adjacent normal tissues
Ans: C

Page
53
6. A patient came to dental clinic with a growth on the inside of the cheek that
is approximately 0.5 cm in diameter and not painful. He noticed the swelling 3
months back. The best treatment is to
a) Remove all growth and send for biopsy
b) Observe periodically
c) Start antibiotic medication
d) Remove part of the growth and send for biopsy
Ans: A
7. Which of the following represents excisional biopsy?
a) Exfoliative cytologic technique
b) Including normal tissue and all of the lesion
c) Including normal tissue and most of the lesion
d) Removing a representative section of the pathological tissue (or) lesion and
some adjacent normal tissue for comparison
Ans: B
8. 5 cm highly suspicious lesion of the floor of the mouth is to studied
microscopically. A specimen is best obtained by
a) Incisional biopsy b) Excisional biopsy
c) FNAC d) None of the above
Ans: A
9. Biopsy specimens removed for examinations are immediately placed in
a) 10% ethanol b) 10% formalin
c) hydrogen peroxide d) 1% formalin
Ans: B
10. A negative histopathoiogy report of a highly suspicious oral lesion suggests?
a) No malignant potential of the lesion
b) Lesion should be stained with toludine blue like stains
c) Periodical recalls are necessary to assess the nature of the lesion
d) That another biopsy is necessary in view of the clinical impression
Ans: D

Page
54
TEMPORO MANDIBULAR JOINT

1. Commonest cause of TMJ ankylosis is


a) trauma b) development disturbances
c) infection d) atrophy
Ans: A
2. Condylar hypoplasia is seen
a) Down’s syndrome b) Goldenhar’s syndrome
c) Grinspan syndrome d) Crouzen disease
Ans: B
3. Ramu, an 8-year-old child fell on his chin and suffered trauma. When he
opens his mouth, it deviates towards left, then the possible condition is
a) Unilateral left TMJ ankylosis b) Unilateral right TMJ ankylosis
c) Bilateral ankylosis d) None
Ans: A
4. Extra articular ankylosis occurs due to
a) # condyle b) # coronoid process
c) fibrosis d) None of the above
Ans: B
5. Hypertrophy of the mandibular condyle may cause the following
a) An anterior cross bite b) Ipsilateral posterior open bite
c) Unilateral class III malocclusion d) All of the above
Ans: D
6. Which surgical procedure is carried out to treat TMJ ankylosis?
a) Arthroplasty b) Condylectomy
c) Discoplasty d) Capsulorrhaphy
Ans: A
7. Early movement of TMJ following surgery for TMJ ankylosis is
a) desirable b) harmful
c) contraindicated d) B & C
Ans: A
8. In dislocation of the jaw, displacement of the articular disc beyond the
articular tubercle of the temporomandibular joint results from spasm or
excessive contraction of the following muscle?
a) Buccinator b) Lateral pterygold
c) Masseter d) Temporalis
Ans: B
9. Treatment of acute dislocation of TMJ is
a) General Anaesthetia b) Muscle relaxants
c) Mechanical manipulation d) All of the above
Ans: C

Page
55
10. The Risdon approach to the TMJ is a through
a) hemicoronal approach b) retroauricular approach
c) preauricular approach d) submandibular approach
Ans: D
11. Commonest complication with Risdon and submandibular incision is
a) damage to marginal mandibular nerve
b) damage to cervical branch of facial nerve
c) damage to inferior alveolar nerve d) damage to lingual nerve
Ans: A
12. Unilateral TMJ ankylosis is associated with the following features, except
a) Multiple carious teeth
b) Facial asymmetry with fullness on the normal side of mandible
c) Chin deviated towards the affected side
d) Prominent antegonial notch on the affected side
Ans: A
13. Bird face appearance Is seen
a) Unilateral TMJ ankylosis b) Bilateral TMJ ankylosis
c) TMJ dislocation d) Bilateral condylar fracture
Ans: B
14. Etiopathology of ankylosis of TMJ includes
a) Intracapsular condylar fractures b) Extracapsular condylar fractures
c) Both the above d) None of the above
Ans: C
15. The tear of intracapsular articular disc of TMJ causes
a) dislocation b) ankylosis c) chronic pain d) arthritis
Ans: C
16. Intraarticular injection of hydrocortisone is given in arthritis of TMJ to?
a) anesthetize the nerve supply.
b) lubricate the joint and increase the range of movement.
c) decrease the inflammatory response.
d) increase the vascular supply.
Ans: C
17. For an 8-year-old child with TMJ ankylosis, the treatment of choice is
a) Gap arthroplasy b) Condylectomy
c) Gap arthroplasty with costochondral grafting
d) Treatment not required
Ans: C
18. A 9 yr old child had trauma to TMJ and ankylosis. He had undergone
arthroplasty. Which of the following graft is best suited to fill the gap?
a) Allograft b) Xenograft
c) Costochondral graft d) Alloplastic Ans: C

Page
56
19. TMJ dislocation most commonly occurs in which direction
a) Anteriorly b) Posteriorly c) Medially d) Laterally
Ans: A
20. Dislocation is treated by forcing the mandible
a) upwards and backward b) upwards and forwards
c) downwards and forwards d) downward and backward
Ans: D
21. Emineetomy is done for treatment of
a) TMJ ankylosis b) TMJ dislocation
c) Coronoid fracture d) All of the above
Ans: B
22. Submandibuiar incisions are preferred in extraoral ramus osteotomies
mainly for
a) esthetics b) accesibility
c) protection of vital organs d) rapid healing
Ans: B
23. Which of the following surgical approach is advocated for demonstrating
the upper parts of and its meniscus
a) Submandibular approach b) Retromandibular approach
c) Preauricular and auricular approach d) Oral approach
Ans: C
24. Of the following which is not a structural element of the TMJ?
a) Joint cavities b) Condyloid process
c) Sigmoid notch d) Articular disc
Ans: C

Page
57
MAXILLARY SINUS

1. During extraction of a maxillary molar, a root tip is left in the maxillary sinus.
The treatment of choice is:
a) Perform Caldwell-Luc to remove the tip
b) Perform hemimaxillectomy
c) Enlarge opening in the socket area
d) No treatment indicated
Ans: A
2. Which of the following symptoms are related to maxillary sinusitis
a) Tenderness over the involved area b) Post nasal drip
c) Change in phonation d) All of the above
Ans: D
3. Oro antral fistula most commonly occurs during extraction of
a) Maxillary 1st premolar b) Maxillary 2nd premolar
c) Maxillary 1st molar d) Maxillary 3rd molar
Ans: C
4. Head-shaking test is done to diagnose
a) Maxillary sinusitis b) Vertigo
c) Position of root in relation to maxillary antrum
d) Maxillary fracture
Ans: C
5. During extraction of maxillary second molar, a pin point 0.5 mm perforation
is created in maxillary sinus. The appropriate treatment would be to
a) Perform Caldwel! Luc procedure
b) Smoother! the bone and place suture across socket
c) Pack the opening with iodoform gauge and allow to heal by secondary
intention
d) No treatment necessary
Ans: D
6. Mention the blood supply to the flap that has been used to close an oro
antral fistula it the area of tooth is
a) Nasopalatine b) Greater palatine
c) Posterior superior alveolar d) Facial
Ans: B
7. Of the following, where antrostomy is commonly used?
a)To remove high maxillary third molars
b)To expose a tooth root for apicectomy
c)To treat an alveolar abscess
d)Following closure of an oroantral fistula
Ans: D

Page
58
8. Caldwell-Luc operation is indicated in
a) chronic maxillary sinusitis b) fracture floor of the orbit
c) vidian neurectomy d) All
Ans: D
9. Nasal antrostomy usually done from this
a) Middle concha b) Inferior concha
c) Middle meatus d) Inferior meatus
Ans: D
10. Symptoms of fresh oroantral communication include
a)Escape of fluids from mouth to nose on the side of extraction
b)Unilateral epistaxis
c)Excruciating pain in and around the sinus
d)All the above
Ans: D
11. An oro-antral communication during extraction
a)requires no treatment immediately
b)immediate prolapse of antral lining occurs into mouth
c)reflux of fluids into the nasal cavity while drinking can be demonstrated
d)must be closed surgically immediately
Ans: D
12. Berger flap procedure is used for
a) TMJ Ankylosis b) TMJ dislocation
c) Oro antrai communication d) Pericoronitis
Ans: C
13. Which of the following procedures can be used to treat an oro-antral fistula
resultingrior surgery?
a) Buccal mucoperiosteai flap
b) Palatal mucoperiosteal flap
c) Bridge flap
d) Palatal island flap
Ans: B

Page
59
CYSTS AND TUMOURS

1. Needle aspiration of a central bone lesion is useful to


a) rule out a vascular lesion b) to determine thickness of buccal plate
c) to diagnose traumatic bone cyst d) feel for root surfaces
Ans: A
2. Aspirate which is creamy white viscoid with the value of total protein less
than 5.0 gm/100ml is suggestive of?
a) Infected cyst b) Dentigerous cyst
c) Odontogenic keratocyst d) Ranula
Ans: C
3. A dirty white aspirate with a protein estimation of <4gm % is suggestive of
(irt to above question)
a) Ossifying fibroma b) Dentigerous cyst
c) Muco-epidermoid carcinoma d) Odontogenic keratocyst
Ans: D
4. Management of Odontogenic keratocyst involves (irt to above question)
a) Marsupialisation b) Enucleation
c) Enucleation with periphereal ostectomy d) Resection and radiation
Ans: C
5. Carnoys solution is used in the treatment of
a) Odontogenic Keratocyst b) Ameloblastome
c) Dentigerous cyst d) Mucocele
Ans: A
6. Which of the following is not a method of treatment of cysts?
a) Cauterisation b) Marsupialisation
c) Enucleation d) Resection
Ans: A
7. Which of the following is not an advantage of marsupialization?
a) Exposure of very little bone b) Preservation of vital structure
c) Rapid healing d) Conserve surgical structures
Ans: C
8. Of the following which is most frequently indicated treatment for
odontogenic cysts
a) Fulguration b) Enucleation
c) Marsupialization d) Incision & drainage
Ans: B

Page
60
9. During enucleation
a) Concave surface should face the cavity lining
b) Convex surface should face the cavity lining
c) The direction of scrapping changes according to the site of cyst
d) Curettes should not be used during enucleation
Ans: A
10. Enucleation is recommended for which of the following odontogenic
tumor?
a) Ameloblastoma b) Myxoma
c) Squamous odontogenic tumor d) Osteoid osteoma
Ans: C
11. Best treatment of the large cyst
a) Enucleation b) Marsupialisation
c) Marsupialisation followed by Enucleation
d) Enucleation followed by marsupialisation
Ans: C
12. The most appropriate method to differentiate between a dentigerous cyst
and an ameloblastoma is through:
a) Radiographic examination b) Aspiration cytology
c) Microscopic examination d) Clinical features
Ans: C
13. Odontogenic tumors just one cm away from lower border
a) Enbloc resection b) Hemimandibulectomy
c) Enucleation d) None
Ans: A
14. When treated with simple curettage which odontogenic tumour is most
likely to recur
a) Complex odontoma b) Compound odontoma
c) Odontogenic myxoma d) Ameloblastic fibroma
Ans: C
15. The treatment of choice for traumatic bone cyst is
a) Marsupialization and leaving the cyst lining
b) Opening the cavity and inducing bleeding
c) Enucleation of the cyst lining
d) Packing with BIPP and prevent bleeding and clot formation
Ans: B
16. The aspirate from a keratocyst will have
a) a low soluble protein content b) a high soluble protein content
c) cholesterol crystal d) inflammatory cells
Ans: A

Page
61
17. A 25-year-old male patient reports with a bony expansile swelling of the
right body of the man dible & mild paresthesia of the right XDN. OPG shows a
multi locular radiolucency without root resorption. What would be your choice
of next investigation?
a) Excision biopsy b) Aspiration Cytology
c) CT Scan d) PET Bone scan
Ans: B
18. Odontogenic keratocyst is noted for its
a) malignant transformation
b) daughter cysts and high rate of recurrence
c) impacted teeth d) nodal metastasis
Ans: B
19. Treatment of odontogenic keratocyst extending into the mandibular ramus
is
a) Enucleation without curettage
b) Marsupiliazation without curettage
c) Enucleation with peripheral osteotomy d) Enbloc dissection
Ans: C
20. Protein concentration of non-keratinizing cyst fluid is
a) 1-2 gm/dL b) 5-11 gm/dL
c) 10-20 gm/dL d) 0-4 gm/dL
Ans: B
21. Marsupialization is a surgical procedure which is commonly used to treat
a/an
a) cystic odontoma b) periapical cyst
c) large cyst d) oro antral fistula
Ans: C
22. Of the following, which is best method to treat a large radicular cyst of the
maxilla with apical involvement of four vital teeth?
a) Marsupialization b) Aspiration
c) Enucleation and primary closure
d) Enucleation and packing the cavity with BIPP
Ans: A
23. Which of the following is not true of enucleation
a)Primary closure is done usually
b)Sometimes packing can be done with healing by secondary intention
c)Sometimes enucleation is followed by marsupialization
d)Enucleation can be done introral or extraoral approach
Ans: C

Page
62
24. Which of the following methods are used in the treatment of hemangioma
of bone
a) Cryotherapy and presurgical embolization
b) Surgery and radiation therapy
c) Sclerosing agents' cryotherapy, and presurgical embolization
d) All of the above
Ans: D
25. Treatment of solitary langerhans histiocytoma of mandible is by
a) Curettage b) Radiotherapy
c) Chemotherapy d) Osteoid osteoma
Ans: A
26. Enucleation of palatal tumour results in
a) excessive bleeding from nasopalatine vessels
b) tearing of nasal mucosa
c) damage to nasopalatine nerve d) alteration of speech
Ans: B
27. Which of the following surgery is indicated for the removal of the 3cm
ameloblastoma in-volving the inferior border of the mandible
a) Enucleation b) Fulguration
c) Cryosurgery d) Resection
Ans: D
28. Odontoma is treated by
a) Excision b) Resection
c) Curettage d) Radiotherapy
Ans: A
29. The best treatment for cementoma is
a) No treatment b) Endodontic treatment
c) Extraction of the teeth
d) Resection of the involved area
Ans: A
30. An empty cavity in the mandible with no lining is mostly likely to be
a) Aneurysmal bone cyst b) Idiopathic bone cavity
c) Dentigerous cyst d) Keratocyst
Ans: B

Page
63
SALIVARY GLAND

1. Carmalt’s gland is/are


a) Major salivary gland b) Minor salivary gland (retromolar)
c) Minor salivary gland (lingual) d) Taste Buds
Ans: B
2. In the clinical evaluation, the most significant finding of a parotid mass may
be accompanying
a) Slow progressive painless enlargement
b) Nodular in consistency
c) Submental & pre-auricular lymphadenopathy
d) Facial paralysis
Ans: D
3. Frey’s syndrome can occur following
a) Saggital split osteotomy b) Caldwell Luc operation
c) Parotidectomy d) Marsupialization
Ans: C
4. Dilation of the salivary duct secondary to epithelial atrophy as a result of
repeated inflammatory or infectious process is
a) Sialodochitis b) Sialadeniti
c) Sialolithiasis d) Mucocele
Ans: A
5. Submandibular calculus can be removed by
a) Dilation of the duct b) Excision of the opening of duct
c) Removal of the gland
d) Incision of the duct and removal of calculus
Ans: D
6. Submandibular duct is exposed via intraoral approach by incising the
a) buccinators b) mucous membrane
c) masseter d) All of the above
Ans: B
7. During the medial approach for submandibular gland surgery the nerve least
likely to be injured is
a) Facial nerve b) Hypoglossal nerve
c) Glossopharyngeal nerve d) Lingual nerve
Ans: C
8. Excision of the submandibular gland for calculus or tumours is done by
incision below angle of the jaw. Special care should be taken to avoid which
nerve?
a) Ansa cervicalis b) Mandible branch of facial nerve
c) Posterior auricular nerve d) Submandibular ganglion Ans: B

Page
64
9. Treatment for mucocele is
a) Marsupialization of the lesion
b) Enucleatin of only the mucocoele without involving the duct of mucous glan
c) Enucleation of mucocele and involving the mucous glan
d) Enucleation of mucocele, involving the mucous gland and surrounding tissue
Ans: D
10. Which of the following detects salivary gland duct calculi?
a) Ultrasonography b) Plain radiography
c) Sialography d) Xeroradiography
Ans: C
11. During surgical excision of the parotid gland the following structures may be
damaged
a) Lesser occipital nerve, hypoglossal nerve, chorda tympani
b) Facial nerve and auriculotemporal nerve
c) Submandibular duct d) Cervical fascia
Ans: B
12. Treatment of pleomorphic adenoma of parotid is
a) Total parotidectomy b) Superficial parotidectomy
c) Deep parotidectomy d) Radical parotidectomy
Ans: B
13. Most common site of sialolithiasis
a) Submandibular duct b) Sublingual duct
c) Parotid duct d) Minor saliavry glands
Ans: A
14. Which of the following is preferred treatment for submandibular salivary
duct stones?
a) Vertical incision, removal of stone and suturing of the incison to close the
duct.
b) Vertical incision and suturing the ductal margins to the oral mucosa leaving it
open.
c) Transverse incision, removal of the stone and closure of the duct.
d) Transverse incision, removal of the stone and suturing the wound, leaving
the duct open.
Ans: B
15. In removal of submandibular gland, which of the following nerves are most
likely to be damaged
a)Facial and glossopharyngeal
b)Lingual and hypoglossal
c)Lingual and glossopharyngeal
d)Hypoglossal and glossopharyngeal
Ans: B

Page
65
16. All the following nerves are encountered during submandibular gland
resection EXCEPT
a) Lingual nerve b) Hypoglossal nerve
c) Accessory nerve d) Marginal mandibular nerve
Ans: C
17. A mucocele of the lower lip should be treated by
a) Incision b) Electro cautery
c) Excision including adjacent minor salivary gland
d) Incision and drainage followed by antibiotics
Ans: C
18. The recurrent ranula is best treated by
a) Eletrosurgery b) Cryosurgery
c) Marsupialization d) Sub-lingual gland excision
Ans: D

Page
66
PREPROSTHETIC SURGERIES

1. The following methods are commonly used methods of ridge extension


procedure of the mandible
a) Secondary epithelialization b) Mucosal graft vestibuloplasty
c) Skin graft vestibuloplasty d) All of the above
Ans: D
2. Skin graft vestibuloplasty prevents relapse by
a) Causing an inhibitory effect on fibroblasts
b) Forms a barrier to reattachment of muscle to periosteum
c) Promoting osteogenesis and bony barrier is established
d) Relapse is common in skin graft vertibuloplasty
Ans: B
3. Myoplasty and sulcus extension procedures are helpful in
a) increasing retention b) increasing stability
c) increasing support d) increasing retention and stability
Ans: D
4. The purpose of alveoloplasty is to
a) Eliminate the undercuts that interfere with seating of the denture
b) Avoid rounding off the sharp bony projections as they will round off in a
few days during the healing process
c) Retain the width of the alveolar ridge even though undercuts exist
d) None of the above
Ans: A
5. All of the following are safely excised in preparing the edentulous for
dentures except
a) pendulous tori b) Genial tubercles
c) Feather edged ridges d) Mylohyoid ridge
Ans: B
6. Of the following conditions where acrylic splints are advisable during the
post operative phase of management
a) Torus palatinus reduction b) Mental tubercle reduction
c) Mandibular alveoloplasty d) None of the above
Ans: A
7. Most tuberosity reductions before denture construction requires
a) primarily soft tissue (or) fibrous tissue removal
b) large amount of bone removal
c) relief from pterygoid hamulus
d) some bony reductions in all instances
Ans: A

Page
67
8. The principal problem with tuberosity reduction is
a) Poor access b) Fomation of oroantral fistula
c) Infection d) Damage to posterior superior alveolar nerve
Ans: B
9. Papillomatosis (or) hyperplastic palatal tissues most often seen beneath
dentures. This condition is best treated by
a) Not allowing the patient to wear the dentures
b) Supraperiosteal dissection (or) electrosurgery
c) Chemical cautery
d) Radical excision because this has malignant potential
Ans: B
10. Which muscle should be detached to lower the floor of the mouth?
a) Mylohyoid b) Geniohyoid
c) Genioglossus d) A & C
Ans: D
11. Submucosal vestibuloplasty is done in
a) maxilla b) mandible
c) maxilla and mandible d) None
Ans: C
12. The soft tissue incision used to remove mandibular torus should be placed
a) Directly over the torus
b) Inferior to the torus in the area of the floor of the mouth
c) Over the edentulous alveolar crest in the gingival crevice around the
dentition
d) A & B
Ans: C
13. Maxillary tori are usually removed by
a) Burs and chisels as indicated to section and remove lobulation
b) Bone forceps to hold torus while it is being sectioned
c) Rongeurs to crush bone d) All of the above
Ans: A
14. Removal of mylohyoid ridge is planned in an edentulous patient. Which of
the following nerves should be protected?
a) Mental b) Mylohyoid
c) Lingual d) Inferior alveolar
Ans: C

Page
68
15. If retromolar pad and tuberosity contact, what should be done?
a) Surgical reduction of tuberosity
b) Do not extend dentures to retromolar area
c) Denture fabrication not possible
d) Has no effect on denture
Ans: A
16. To remove an undercut maxillary tuberosity, the following flap design
criteria is true
a) Incision should be made at the crest of the ridge
b) A “wedge” should be done first
c) Incision may extend to premolar canine area
d) All of the above
Ans: D
17. Among the following which is a problem with reconstruction of the
atrophic ridge with hydroxyl apatite granules
a) Displaced material b) Mental nerve dysesthesia
c) Poor ridge form d) Any of the above
Ans: D

Page
69
IMPLANTS AND MISCELLANEOUS

1. Osseointegrated implants have the following features


a) They form junctional epithelium with the surrounding tissues.
b) They are anchored directly to living bone as determined by radiographic and
light microscopic analyses.
c) They have direct structural and functional connection with bone only at the
radiographic level of detection.
d) They form a pseudo-periodontal ligament.
Ans: B
2. Which of the following statement about the bone quality is untrue?
a) D4 bone is the densest b) Dl bone is the densest
c) There is a direct correlation between bone density and implant survival
d) Bone quality is determined precisely based on Hounsfield numbers
Ans: A
3. The minimum inter-space for fixed implant supported prosthesis is
a) 4 mm b) 6 mm c) 7 mm d) 12 mm
Ans: A
4. Which of the following is true in regard to reconstruction of an edentulous
mandible with implants
a) Subperiosteal implant requires only single surgery.
b) Staple implant is most useful for the posterior mandible.
c) Both blade and the osteointegrated cylinder are useful as posterior abutments
in patients with high mental foramen,
d) Osteointegrated concept of implant. Stabilisation has the best documentation
of long-term success.
Ans: D
5. Minimum mesiodistal width required for the placement of an implant of
diameter 4.1 mm is
a) 4 mm b) 6 mm c) 7 mm d) 9 mm
Ans: C
6. Important diagnostic tool for achieving accurate implant angulation
a) Diagnostic template b) Wax up
c) Mounted diagnostic cast d) Surgical template
Ans: D
7. During implant placement the maximum allowed threshold of temperatures
raise in centigrades is
a) 37°C b) 47°C c) 57°C d) 67°C
Ans: B

Page
70
8. Which of the following is true in regard to reconstruction of an edentulous
mandible with implants
a) Subperiosteal implant requires only single surgery.
b) Staple implant is most useful for the posterior mandible.
c) Both blade and the osteointegrated cylinder are useful as posterior abutments
in patients with high mental foramen.
d) Osteointegrated concept of implant. Stabilisation has the best documentation
of long term success.
Ans: D
9. Which of the following has greater bone density
a) Post mandible b) Post maxilla
c) Anterior maxilla d) Anterior mandible
Ans: D
10. What is the minimum distance between the nasal vestibule and implant?
a) 1 mm b) 2 mm c) 2.5 mm d) 3 mm
Ans: A
11. Among the following which indicates successful allogenic tooth implant?
a) The tooth survives for 12 months.
b) The periodontal ligament is re-established.
c) Both the pulp and periodontal ligament survive.
d) Ankyiosis occurs
Ans: D
12. A direct connection between living bone and load-bearing endosseous
implant at the light micro scopic level
a) Osteosynthesis b) Osteogenesis
c) Osseointegration d) Osseoinduction
Ans: C
13. The amount of rate of distraction in osteogenesis distraction surgeries
a) l mm /day b) 2.5 mm /day
c) 5 mm /day d) 10 mm /day
Ans: A
14. Latency period in distraction osteogenesis of mandible is
a) 5 to 7 days b) 6 months
c) 2 weeks d) 3 months
Ans: A
15. Cryosurgery utilizes which of the following?
a) Nitrous oxide b) Carbon dioxide
c) Liquid oxygen d) Nitric dioxide
Ans: A

Page
71
16. During cryosurgery
a) Cells get evaporated b) Cells will not die, only freeze
c) Cell death occurs when the temperature falls below – 20°C
d) Patients need general anesthesia
Ans: C
17. Among the following which are characters of an ideal graft
a) It should withstand mechanical forces.
b) It should produce no immunological response.
c) It should actively assist osteogenic process of the host.
d) All of the above
Ans: D
18. ………………is best to restore a missing mandibular portion
a) Homogenous bone b) Autogenous bone
c) Silastic d) Tantalum
Ans: B
19. A 30yr old patient underwent resection of mandible for the treatment of
ameloblastoma. The best graft is
a) Free iliac crest graft b) Free vascularized iliac crest graft
c) 6th rib d) Allograft of the bone
Ans: B
20. Bone transplant from one human to another is termed
a) Allogenous b) Autogenous
c) Homologous d) Heterogenous
Ans: C
21. For an elective surgery the minimum hematocrit value should be
a) 40 b) 30 c) 20 d) 10
Ans: B
22. The thickness of split thickness skin graft used in maxillofacial surgery ranges
from
a) 2 to3 mm b) 1 to 2 mm
c) 0.3 to 0.5 mm d) 1.3 to 1.5 mm
Ans: C
23. Generally bone marrow for grafting the defects is obtained from
a) The iliac crest b) The mandible
c) Maxillary tuberosity d) Rib
Ans: A
24. Among the following which is not used in the fixation of bone grafts
a) Bone plates b) Titanium mesh
c) Gut d) Tranosseous wires Ans: C

Page
72
TRAUMATOLOGY

1. A patient with maxillofacial injuries should be carried in


a) supine position b) lateral position
c) prone position d) sitting position
Ans: B
2. The presence of tension pneumothorax should be considered in patients who
has feature as
a) Development of severe respiratory distress with decreased breath sounds,
hyper resonance on one side of chest
b) Development of severe respiratory distress and deviation of trachea to the
side involved
c) Patient becomes acutely ill, with collapsed neck veins.
d) All of the above
Ans: A
3. Raised intracranial pressure following a maxillofacial injury
a) is a normal finding
b) is typically associated with an increase in the heart rate
c) is typically associated with an increase in the blood pressure
d) usually subsides spontaneously
Ans: C
4. Which of the following is an immediate danger to a patient with severe
facial Injuries
a) Bleeding b) Associated fracture spine
c) Infection d) Respiratory obstruction
Ans: D
5. Which of the following always indicates obstruction to the airway?
a) Slow pounding pulse b) Stertorous breathing
c) Increase in pulse rate d) Decrease in blood pressure
Ans: B
6. The head tilt procedure while dealing with an unconscious patient in dental
chair is done to ensure
a) patent airway b) blood circulation to the brain
c) to clear the foreign body obstacle
d) to relieve spasm of respiratory muscles
Ans: A
7. Of the following, which is the immediate treatment for a patient with
comminuted fracture and in the state of shock?
a) Ringer’s lactate solution by IV route b) Normal saline by IV route
c) Blood transfusion d) Plasma expanders
Ans: A

Page
73
8. Definitive management of maxillofacial trauma after trauma can be delayed
up to what time without substantially compromising the outcome?
a) Should be done 5-7 days of admission
b) Can be delayed up to 2 weeks
c) Should be performed immediately after admission d) None
Ans: B
9. Clinical sign that is always positive in bone fracture is
a) Crepitus b) Tenderness
c) Abnormal mobility d) All of the above
Ans: B
10. Direct impact on the bone will produce a
a) Transverse fracture b) Oblique fracture
c) Spiral fracture d) Comminuted fracture
Ans: A
11. Green stick fractures are most common with a………...
a) Older people b) Adult
c) Children d) Soldiers
Ans: C
12. Osteomyelitis is most commonly associated with………...
a) compound fractures b) comminuted fractures
c) green stick fractures d) telescopic fracture
Ans: A
13. Principles in treating fractures include
a) Reduction of fracture
b) Fixation of fracture and restoration of occlusion
c) Immobilisation d) All of the above
Ans: D
14. During panfacial fractures the structures that are reconstructed first are
a) Outer bone segments of the frame b) Bones inside the framework
c) Maxilla always d) Mandible always
Ans: A
15. The most common site of fracture of the mandible is the
a) body b) angle c) symphysis d) condyle
Ans: B
16. The weakest part and most vulnerable part for fracture in the mandible is
a) Ramus of mandible b) Condylar neck
c) Angle of the mandible d) Region of the canine tooth
Ans: B
17. Bones usually fracture at the site of
a) Compressive strain b) Tensile strain
c) Rich blood supply d) Thin periosteal covering Ans: B

Page
74
18. Orbital blow out fracture is best seen in?
a) Reverse town b) Waters view
c) PA view d) Submentovertex
Ans: B
19. The radiographic view of choice for diagnosing horizontally
favorable/unfavorable # is:
a) OPG b) Occlusal
c) Lateral Oblique view d) Transpharyngeal
Ans: C
20. Condylar neck fracture best detected in
a) Submentovertex projection b) Water’s projection
c) Posterio anterior skull projection d) Reverse towne projection
Ans: D
21. Of the following which is almost pathognomonic of a mandibular fracture
a) Deep laceration near the area of trauma and paresthesia
b) Ecchymosis in the lingual sulcus and dearranged occlusion
c) Anterior open bite and deviation of the mandible on opening
d) None of the above are correct. There is no pathogenic feature of mandibular
fracture
Ans: B
22. All are features of mandibular fracture except:
a) Malocclusion b) Paresthesia of lower lip
c) Fractured ends are prevented from dislocation by masticatory muscles
d) Are usually compound
Ans: C
23. The proximal segment of mandible angle fracture usually displaced in
a) Anterior and superior b) Posterior and interior
c) Interior only d) Posterior and superior
Ans: A
24. In a patient with bilateral dislocated fractures of the necks of the
mandibular condyles, one can expect the following clinical signs
a) Anterior open bite b) Inability to protrude the mandible
c) Inability to bring posterior molars into contact d) A & B
Ans: D
25. Which of the following are most complicated fractures?
a) Symphysis b) Body c) Condyle d) Angle
Ans: C

Page
75
26. A 7-year-old boy presented with fracture of left sub condylar region with
occlusion undisturbed, the treatment would be
a) Immobilization for 7 days
b) Immobilization for 14 days with intermittent active opening
c) No immobilization with restricted mouth opening for 10 days
d) No immobilization and no active treatment
Ans: D
27. In children fracture mandible which has not been perfectly reduced but has
become firm with slight imperfection, the choice of the management would be
a) Arch bar and elastric traction
b) Accept the slight imperfection and allow for later remodeling
c) Refracture and plating d) Refracture and interosseous wiring
Ans: B
28. An adult patient sustained a subcondylar fracture on the left side. Clinically
it is seen that there is
a) Moderate intraoral bleeding b) Trismus and bilateral crepitus
c) Deviation of the mandible to the right-on protrusion
d) nability to deviate the mandible to the right-on opening
Ans: D
29. The muscle, under the influence of which, the superior fragment of
condyle, in a condylar neck # is displaced anteriorly and medially is
a) Medial Pterygoid b) Lateral Pterygoid
c) Masseter d) Temporalis
Ans: B
30. In case of pure symphysis fracture of mandible, fracture segments are
usually displaced
a) Lingually & downward by the pull of Genioglossus mylohyoid muscles.
b) Mainly lingually and lingual & downward movement due to the pull of
Geniohyoid & myohyoid muscle.
c) Mainly upward movement. Lingual and upward movement due to the pull
of Geniohyoid and mylohyoid muscle.
d) None of the above are correct. A little (or) no displacement occurs.
Ans: D
31. Posterior displacement of the fractured anterior segment in the bilateral fracture
of the mandible in the canine region is due to the action of the
a) Thyrohyoid, genioglossus and geniohyoid
b) Mylohyoid, genioglossus and geniohyoid
c) Geniohyoid and genioglossus and anterior belly of digastric muscles
d) Mylohyoid, geniohyoid and thyrohyoid muscle
Ans: C

Page
76
32. A fractured coronoid process of mandible displaces upwards by the action
of following muscle
a) Buccinator b) Lateral Pterygoid
c) Mylohyoid d) Temporalis
Ans: D
33. Which of the following is seen with guardsman fracture associated with
bilateral condylar fractures
a) Increased intercanthal distance b) Increased interpupillary distance
c) Increased gonion gnathion distance
d) Increased interangular distance
Ans: D
34. Fracture most commonly resulting in widening of face is?
a) Parasymphyseal and symphyseal region fracture without condylar fracture
b) Parasymphyseal and symphyseal region fracture with condylar fracture
c) Zygomaticomaxillary complex fracture
d) Cranio-facial disjunction fracture
Ans: B
35. Fracture of mandible not involving dental arch is treated by
a) Open reduction b) Closed reduction
c) No treatment required d) None of the above
Ans: A
36. A displaced, unfavorable fracture in the mandibular angle region is a
potentially difficult fracture to treat because of
a) Injury to neurovascular bundle
b) Malocclusion secondary to injury
c) Distraction of fracture segments by muscle pull
d) Increased density of bone in this region of mandible
Ans: C
37. Unfavorable fracture of angle of mandible is best treated by
a) Closed reduction with IMF b) Closed reduction with cap splint
c) Open reduction with bone plate fixation
d) Circummandibular wiring
Ans: C
38. The splint which is most commonly used in dentulous mandibular fracture
is………
a) Gunning splint b) Cap splint
c) Ribbon splint d) All of these
Ans: B
39. How many weeks of fixation are required for fracture mandible?
a) 8-10 weeks b) 6-8 weeks
c) 4-6 weeks d) 2-4 weeks Ans: C

Page
77
40. Which of the following is used for ‘figure of eight’ wiring for stabilization of
subluxated teeth in alveolar injuries?
a) 0.23 mm stainless steel wire b) 0.35 mm stainless steel wire
c) 0.45 mm stainless steel wire d) 0.55 mm stainless steel wire
Ans: B
41. A fracture of the mandible in the canine region in a 6-year-old child should
be managed by
a) Cap splint fixation b) Intermaxillary fixation
c) Risdon wiring d) Transosseous wiring
Ans: A
42. Which one of the following is not an appropriate method of stabilizing # in
childhood
a) Arch bar with circumdental wiring
b) Circumdental wiring with skeletal wiring
c) Risdon wiring d) Splints
Ans: A
43. The movement employed in the reduction of dislocated non-fractured
mandible condyle is
a) Downward and backward b) Upward and forward
c) Upward and backward d) Downward and forward
Ans: A
44. Paresthesia is one of the commonest finding in which of the following
fractures?
a) Subcondylar fracture of the mandible
b) Zygomatico maxillary complex fracture
c) Fractured coronoid and displacement of the fracture
d) Symphysis fracture associated with bilateral subcondylar fracture
Ans: B
45. Paresthesia over nasal and upper lip following fractured zygoma is because
of involvement of
a) Facial nerve b) Posterior superior alveolar nerve
c) Infraorbital nerve d) Mental nerve
Ans: C
46. A 35-year-old man with a Le Fort III fracture complains of blood tinged
watery discharge from his nose 2 days after the trauma. CT scan confirms NOE
fracture as well. What clinical features suggest that the discharge is CSF Leak?
a) Continuous discharge
b) Appearance of tram line or halo rings
c) Stoppage of discharge on pressure
d) None of the above
Ans: B

Page
78
47. The most common complication of CSF Rhinorrhea is (irt to above
question)
a) Brain herniation b) Blindness
c) Ascending Meningitis d) Cavernous sinus thrombosis
Ans: C
48. The most common site of leak in CSF rhinorrhoea is
a) Sphenoid sinus b) Frontal sinus
c) Cribriform plate d) Tegmen tympani
Ans: C
49. CSF rhinorrhea is not seen in
a) Le Fort I b) Le Fort II
c) Le Fort III d) Ethmoidal
Ans: A
50. Le Fort III fracture is the same as
a) Craniofacial disjunction b) Guerrin’s fracture
c) Pyramidal fracture d) None of the above
Ans: A
51. Floating maxilla is typically found in
a) Le Fort I or Guerin fractures
b) Le Fort II or pyramidal fractures
c) Craniomandibular dysjunction
d) All of the above
Ans: A
52. In Le Fort I fracture, the fracture fragment includes all of the following
except the
a) bridge of the nose b) nasal spine
c) lower portions of the pterygoid d) upper teeth and palate processes
Ans: A
53. Which of the following is characteristic of Le Fort I fracture?
a) CSF rhinorrhea b) Bleeding from the ear
c) Bleeding into the antrum d) A & B
Ans: C
54. “Panda Facies” is one of the term to describe the patient’s face after mid
face trauma. The appearance is due to
a) Gross swelling of the face
b) C.S.F. rhinorrhea and bleeding from the nose & laceration results in red &
white streaks on the face
c) Edema & ecchymosis around the eyes
d) Sub-conjunctival hemorrhage (bilateral)
Ans: C

Page
79
55. The Sub-conjunctival hemorrhage remains red in color for a long time
because?
a) Hemoglobin prevents breakdown of R.BC.
b) Permeability of the conjunctiva to oxygen
c) Oxygen content of blood to conjunctiva is vey high
d) All of the above
Ans: B
56. Bilateral subconjunctival ecchymosis is not associated with
a) Le Fort II fracture b) Le-Fort III fracture
c) Naso ethmoidal complex fracture d) Le-Fort I fracture
Ans: D
57. The incision used for treatment of traumatic telecanthus is
a) Bicoronal incision b) Upper blepheroplasty
c) Infraorbital incision d) Preauricular incision
Ans: A
58. In Le Fort III fracture which of the following are seen
a) Whole face is mobile with crack pot sound on tapping teeth
b) CSF rhinorrhea
c) Fracture at frontozygomatic suture
d) All of the above
Ans: D
59. Battle sign is seen in
a) # of anterior cranial fossa b) # of middle cranial fossa
c) # of posterior crania! Fossa d) None of the above
Ans: B
60. Of the following which is weakest part of orbit
a) Medial wall b) Lateral wall
c) Floor of the orbit d) Roof of the orbit
Ans: A&C
61. Hanging drop effect in blow-out fracture is due to herniation of
a) Superior oblique and inferior oblique muscles
b) Inferior oblique and inferior rectus muscles
c) Superior rectus and superior oblique muscles
d) Inferior oblique and superior rectus muscle
Ans: B
62. Diplopia is most common with
a) Mandibular body fracture b) Alvedan fractures
c) Craniofacial dysjunction d) Zygomatico maxillary complex
Ans: D

Page
80
63. After orbital injury the cause of diplopia in the acute phase is?
a) Ischemic injury leading to fibrosis
b) Entrapment of extra ocular muscles
c) Intrusion of eye d) None
Ans: A
64. Which of the following statements about a blow-out fracture are true?
a) It has been established beyond doubt that such an injury is caused by the
eyeball being forced backwards, thus raising the intraorbital pressure and
causing the thin orbital floor to blow-out.
b) If the patient has diplopia and a coronal tomogram shows a small blow-out
of the orbital floor, it should be repaired immediately.
c) Early repair of the orbital floor always prevents enophthalmos
d) None of the above
Ans: D
65. In a patient with fresh blowout fracture of the orbit, best immediate
management is
a) Wait and watch b) Antral pack
c) Titanium mesh d) Glass bead mesh
Ans: A
66. Gillis approach for reduction of zygomatic fracture is done through
a) Temporal fossa b) Intra temporal fossa
c) Infra orbital fossa d) All of the above
Ans: A
67. In depressed zygomatic arch fracture, difficult in opening the mouth is
caused by impingement of
a) Condyles b) Ramus
c) Petrous temporal d) Coronoid process
Ans: D
68. After fracture of middle cranial foramen there is epiphora this is due to
damage of
a) Ciliary ganglion b) Greater palatine nerve
c) Infraorbital nerve d) Nasolacrimal duct
Ans: D
69. All of the following statements of nasal fractures are true except
a) Even if minor, they may be followed by bilateral ecchymosis and facial
oedema
b) They may need to be reduced for a few weeks
c) They need not be complicated by traumatic telecanthus
d) They may lead to the telescoping of the nasal complex into the frontal sinus
Ans: A

Page
81
70. Which of the following statements about CSF rhinorrhoea is true?
a) It suggests a fracture of the petrous temporal bone
b) It reduces after a Valsalva manoeuvre
c) The fluid tastes sweet
d) None of the above
Ans: D
71. Which of the following methods of osteosynthesis in the treatment of
fracture mandible results in primary bone healing?
a) Mini-plates b) Intraosseous wiring
c) Inter-maxillary fixation d) Compression bone plates
Ans: D
72. In osteosynthesis all are used except
a) Lag screw b) Wires c) Clampy bone plate d) Eyelet wiring
Ans: D
73. Miniplate fixation is effective at
a) Zone of tension b) Near the roots of teeth
c) Zone of compression d) Near to the mental foramen
Ans: A
74. Where on a true lag screw are the threads placed?
a) On the distal end, only b) On the end close to the head
c) On the entire length of screw
d) Both on distal end and the end near the head
Ans: A
75. In fracture of atrophic mandible treatment modality is
a) Bone grafting & load bearing b) Bone grafting & load sharing
c) Open reduction d) Semi rigid
Ans: A
76. Mandibular fracture managed by adapting mini plates is a type of
a) Load bearing osteosynthesis b) Load sharing osteosynthesis
c) Non-rigid osteosynthesis d) Rigid osteosynthesis
Ans: B
77. Primary healing of a mandibular fracture is seen following fixation with
a) Gunning splints b) Compression plates
c) Trans-osseous wires d) Champy plates
Ans: B
78. According to the tension and compression forces acting at the condylar
border which of these would best attenuate them?
a) A plate at the anterior border and another at the posterior border
b) A plate at the anterior border
c) A plate at the post border
d) A plate at the lateral border Ans: D

Page
82
79. The optimum length of screw, for fixation of plate in mandible is
a) 2 mm b) 3 mm c) 4 mm d) 6 mm
Ans: C
80. Open reduction and internal fixation of fracture fragments in the older
patients is risky because
a) of their old age, they less likely tolerate the major procedure under general
anesthesia.
b) fixation is difficult because bone will become denser as age advances.
c) delayed (or) non-union may occur because of over all decrease in reparative
abilities of the body.
d) None of the above are correct.
Ans: D
81. Eburnation is seen in
a) Malunion b) Non-union c) Osteomyelitis d) Osteoradionecrosis
Ans: B
82. Which of the following is a complication of an open fracture?
a) Malunion b) Nonunion c) Infection d) Crepitation
Ans: C
83. Cause of death during the 2nd peak in a patient with severe injuries is
a) Spinal injury b) Epidural and Subdural hemorrhage
c) Aortic arch rupture d) Sepsis
Ans: B
84. A patient with suspendcted cervical fracture should be kept in
a) Prone position b) Both body and neck fiexed
c) Body and neck extended d) Body extended and neck flexed
Ans: C
85. The recommendation about the resuscitation of elderly trauma patients
include all except
a) The presence of pre-existing medical conditions will adversely affect the
outcome in elderly trauma patient.
b) Elderly trauma patients (65 years or older) with a GCS score less than 8 has a
very poor prognosis. If improvement is not possible within 72 hours, then
limitation of continued aggressive therapy should be considered.
c) Invasive hemodynamic monitoring is not indicated for those geriatric trauma
patients even with uncertain cardiovascular or renal diseases.
d) Effort should be made to optimize the cardiac index (>4 L/min./m2) and
oxygen consumption index (170 mL/min/m2)
Ans: C

Page
83
86. Gunshot wounds of the face caused by
a) Low-veiocity hand-gun bullets are typically less serious than those caused by
high velocity missiles.
b) Low-velocity missiles have a deceptive external appearance in that there is
extensive internal damage covered by a small entry wound.
c) High-velocity bullets are sterile.
d) High-velocity bullets must be sutured as soon as possible.
Ans: B
87. From the scheme given below, choose the appropriate order of priorities in
the management of a patient with polytruama
1. Control of extermal haemorrhage
2. Intravenous infusion and transfusion
3. Maintenance of a patient airway
4. Relief of a tension pneumothorax
5. Splinting of fractures
a) 1, 5, 2, 3, 4 b) 3, 4, 1, 2, 5
c) 4, 1, 5, 3, 2 d) 2, 1, 5, 3, 4
Ans: B
88. The first step in management of head injury is
a) Secure airway b) I.V. mannitol
c) I.V. dexamethasone d) Blood transfusion
Ans: A
89. The safest initial approach to open airway of patient with maxillofacial
trauma is
a) Head tilt – chin tilt b) Jaw thrust technique
c) Head lift – neck lift d) Heimlich procedure
Ans: A
90. An average patient with maxillofacial trauma requires how much of daily
sodium?
a) 100 mmol b) 50 – 60 mmol c) 10 mmol d) 1000 mmol
Ans: A
91. Suturing in facial would injuries should be done within
a) 2 hours b) 6 hours c) 4 hours d) 8 hours
Ans: B
92. The fracture of the tooth bearing segment of the mandible is
a) simple b) complex c) compound d) comminuted
Ans: C
93. Fractures in edentulous mandible are commonly described as
a) Compound fracture b) Comminuted fracture
c) Simple fracture d) Compound comminuted fracture
Ans: C

Page
84
94. Of the following which facial bone is most frequently fractured?
a) Mandible b) Maxilla c) Nasal d) Zygomatic
Ans: C
95. Which side of face is more commonly affected in trauma?
a) Right b) Left c) Both equall d) None
Ans: B
96. The most common anatomic site of fracture of mandible is the
a) body b) angle c) condyle d) symphysis
Ans: B
97. Zygomatic fracture is better viewed in
a) Reverse town b) Waters view
c) PA view d) Submentovertex view
Ans: D
98. The radiographic view of choice for demonstrating a nasal # is:
a) Reverse towne’s b) PA view
c) True lateral d) OPG
Ans: C
99. Fracture of mandible all are true except
a) Fractures of the mandible are common at the angle of the mandible.
b) Fractures of the mandible are effected by the muscle pull.
c) Fractures of the mandible are usually characterized by sublingual hematoma.
d) C.S.F. rhinorrhea is a common finding.
Ans: D
100. A patient came to the trauma center who had a blow over his lower jaw.
Intraoral examination reveals hematoma near lingual side of lower second
molar. The proximal fragment is medially displaced. Which of the following
might explain the above findings?
a) Vertically favorable fracture of angle of the mandible & displacement is due
to masseter action
b) Vertically unfavorable fracture of angle of the mandible & displacement is
due to internal pterygoid action
c) Horizontally favourable fracture at the angle & displacement is due to medial
pterygoid action
d) Horizontally unfavourable fracture at the angle & displacement is due to
masseter action
Ans: B
101. After a blow to the chin, patient has pain and tenderness over the right
TMJ. Open bite, and deviation of the chin to the right side. This suggests
a) Fracture mandible right angle region b) Fracture mandible left angle region
c) Fracture mandible right condyle d) Fracture mandible left condyle
Ans: C

Page
85
102. Diagnostic findings of condylar fractures include which of the following
a) Limitation in opening mouth
b) Evidence of facial trauma, especially in the areaof mandible and symphysis
c) Deviation, upon opening, towards the involved side
d) All of the above
Ans: D
103. Early complication following fracture of the mandibular condyle includes
all except
a) Fracture of tympanic plate
b) Vascular injuries
c) Fracture of glenoid fossa with or without displacement of the condylar
segment into the middle cranial fossa
d) Malocclusion
Ans: D
104. In which of the following anterior open bite occur?
a) Bilateral condylar fractures
b) Symphysis fracture on one side and angle fracture on the other side of the
mandible
c) Horizontal fracture of the maxilla
d) A & C
Ans: D
105. Sixteen-year-old boy presented with pain on left side of the TMJ on
mandibular movement. Patient gives H/O road side accident one day back.
Clinically, occlusion is normal. Radiograph reveals undisplaced crack in the left
mandibular condyle. What is the treatment of choice?
a) Open reduction & internal fixation
b) Maxillo-mandibular fixation for 3 weeks
c) Maxillo-mandibular fixation for 4 weeks
d) No surgical intervention, instruction for soft diet for 3 weeks and
observation for any change in occlusion
Ans: D
106. Most common complication of condylar injuries in children
a) Pain b) Ankylosis
c) Osteoarthritis d) Fracture of glenoid fossa
Ans: B
107. In case of subcondylar fracture, the condyle moves in
a) Anterior lateral direction b) Posterior medial direction
c) Posterior lateral direction d) Anterior medial direction
Ans: D

Page
86
108. In bilateral parasymphysis fracture the mandible is pulled posteriorly due
to the action of
a) Genioglossus b) Geniohyiod
c) Mylohyoid d) Hyoglossus
Ans: A
109. Excessive muscular contraction is one of the frequent cause of
a) Unilateral condylar fracture b) Coronoid fracture
c) Fracutre of angle of the mandible d) Bilateral condylar fracture
Ans: B
110. Facial widening/flaring is a complication seen in
a) Symphyseal # b) Bilateral condylar #
c) Tightening of rnaxillo-mandibular d) All of the above fixation
Ans: D
111. Fracture of body of mandible with full arch of teeth [undisplaced] is treated
by
a) IMF b) Open reduction and internal fixation
c) Close reduction and internal fixation
d) External pin fixation
Ans: A
112. The treatment for a mandibular fracture between the incisors is treated
with
a) Risdon wiring b) Essig wiring
c) Cap splint with circum-mandibular wiring d) Transosseous wiring
Ans: A
113. The ideal treatment for fracture of the angle of mandible is
a) Transosseous wiring b) Intermaxillary fixation
c) Plating on the lateral side of the body of the mandible
d) Plating at the inferior border of the mandible
Ans: D
114. In condylar fracture with greater than 5 mm overlapping of greater with
fracture segment, what is the line of treatment?
a) Closed reduction and IMF b) ORIF
c) Soft diet d) No treatment
Ans: B
115. A Gunning’s splint is used in the treatment of
a) Fracture of the condyiar neck of a child
b) Fracture of the edentulous mandible
c) Anterior dentoalveolar fracture
d) None of the above
Ans: B

Page
87
116. Treatment period of fixation for alveolar bone fracture is
a) 3-4 week b) 6-8 week
c) 2-3 week d) 1 week
Ans: A
117. Important factor in treatment of condylar neck fractures in young patients
is by
a) Early mobilization b) Surgical approach
c) Minibone plating d) All the above
Ans: A
118. In children with mixed dentition fracture mandible is treated by
a) Bone plating b) Transosseous wiring
c) Eyelet intermaxillary wiring d) Splinting with circumferential wiring
Ans: D
119. After reduction of a mandibular dislocation that occurred for the first time,
treatment should be to
a) Inject sclerosing solution into the joint, so further dislocation is less likely.
b) Inject corticosteroids into the joint which reduces the inflammation.
c) Immobilize with IMF for 5-6 weeks.
d) Advise the patient to limit opening of the mouth for 2-3 weeks.
Ans: D
120. Facial paralysis following maxillofacial fractures is most common in
a) # of the condylar neck b) # of the nasal bones
c) # of the zygomaticomaxillary d) # of the mandibular symphysis complex
Ans: A
121. The inferior orbital fissure is the key to remembering the usual lines of
ZMC #, three lines of fracture extend from inferior orbital fissure are
a) Antero-medial, a supero-medial and lateral direction
b) Antero-medial, a supero-lateral and inferior direction
c) Antero-inferior, a supero-posterior and medial direction
d) Antero-lateral, a supero-lateral and medial direction
Ans: B
122. Flattening of the cheek seen in
a) Maxillary fractures b) Condylar fractures
c) Zygomatic bone fractures d) Orbital injuries
Ans: C
123. CSF rhinorrhea is found in
a) Frontal bone structure b) Zygomatico maxillary fracture
c) Naso ethmoidal fracture d) Condylar fracture
Ans: C

Page
88
124. The most common site of leak in CSF rhinorrhea is into
a) Sphenoid sinus b) Frontal sinus
c) Ethmoidal sinus d) Maxillary sinus
Ans: C
125. Fracture commonly pyramidal in shape is
a) Le Fort 1 b) Le Fort 2
c) Le Fort3 d) Mandibular
Ans: B
126. Craniofacial dysjunction seen in
a) Le Fort III fracture b) Guerrin’s fracture
c) High Le Fort I fracture d) None of the above
Ans: A
127. Moon face is seen in
a) Le Fort I b) Le Fort II
c) Le Fort III d) Orbital fractures
Ans: B
128. Which of the following is not a feature of Le Fort II fracture?
a) Enophthalmos b) Malocclusion
c) Paraesthesia d) CSF rhinorrhea
Ans: A
129. Sub-conjunctival haemorhage occurs due to
a) Zygomatic complex fractures b) Orbital fractures
c) Le Fort II fractures d) All of the above
Ans: D
130. The intercanthal distance suggestive of traumatic telecanthus is
a) 30 mm b) 32 mm c) 25 mm d) 40 mm
Ans: D
131. “Dish face’ deformity commonly seen with fractures of middle third of face
is because of
a) Posterior and downward movement of maxilla
b) Anterior and forward movement of maxilla
c) Anterior and downward movement of maxilla
d) Nasal complex fracture
Ans: A
132. Hooding of eyes is seen in which fracture
a) Le Fort I b) Le Fort II c) Le Fort III d) Nasal bone
Ans: C
133. Ecchymosis in the post auricular region over the mastoid process is
called as
a) Battle’s sign b) Murphy’s sign c) Guerin’s sign d) None
Ans: A

Page
89
134. “Hanging drop” sign in the maxillary sinus radiograph usually indicates
a) Nasal bone fracture
b) Orbital floor blow out fracture
c) Isolated coronoid fracture fragment hanging by temporalis muscle
d) Condylar fracture (see expln)
Ans: B
135. In blowout fracture of the orbital floor the following occurs
a) Paralysis of lower eyelid b) Paralysis of upper eyelid
c) Loss of sensation of upper lip d) None of the above
Ans: C
136. Diplopia resulting due to facial trauma is all except
a) Monocular diplopia usually indicates a detached lens, hyphema or other
traumatic injuries of globe requiring immediate attention of ophthalmology.
b) Binocular diplopia as a result of trauma due to soft tissue entrapment
(muscle or periorbital), neuromuscular injury or a change in orbital shape.
c) General edema of the orbit usually causes diplopia in the extremes of medial
or lateral gaze.
d) Diplopia of edema & hemorrhage origin should resolve in a few days,
whereas due to entrapment of orbital tissue does not.
Ans: C
137. Basic mechanism responsible for post traumatic enopthalmos
a) Increase in size of orbit
b) A decrease in content of orbit
c) A disruption in the ligament structure of the globe
d) All of the above
Ans: D
138. In blow out fractures which of the following is seen
a) Enophthalmos b) Exophthalmos
c) Bulbar hemorrhage d) None
Ans: A
139. Depressed fracture of the zygomatic area may be clinically recognised by
a) Concavity of the overlying tissue in the zygomatic arch area
b) Interference with movements of the mandible
c) Subluxation of condyles
d) A & B
Ans: D
140. During the Gillies approach, the structure of anatomic significance is
a) Superficial temporal artery b) Marginal mandibular nerve
c) Internal jugular vein d) Inferior alveolar nerve
Ans: A

Page
90
141. Fixation with pack in maxillary sinus is
a) To support comminuted fracture of the body of zygomatic complex
b) To support and reconstitute comminuted orbital floor fracture
c) To protect mucosal covering of maxillary sinus
d) [A] and [B] are correct
Ans: D
142. The muscle that aids in displacement of maxillary fractures are
a) Masseter b) Temporalis
c) Orbicularis oculi and orbicularis oris d) None
Ans: D
143. Key to successful open reduction of nasal fractures is?
a) Patient presenting before 4 weeks (early treatment)
b) Management of the nasal septum
c) Cartilage segment of septum be handled without resection at all the times
d) All the above
Ans: B
144. Primary bone healing all are true except
a) Gap healing and contact healing both produce healed fracture without
intermediate cartilaginous callus formation
b) Contact healing is process of bone formation occurring when the
interfragmentary gap is essentially zero
c) If the interfragmentary gap is less than 0.5 mm lamellar bone forms directly
d) Primary bone healing relies heavily on periosteum for repair
Ans: D
145. Champy’s plates are?
a) Rigid fixation with bicortical screws
b) Semirigid fixation with monocortical screws
c) Rigid fixation with monocortical screws
d) Semirigid fixation with bicortical screws
Ans: B
146. Which of them is not rigid osteosynthetic fixation?
a) Osteosynthesis b) Microplating
c) Screw plating d) Wiring
Ans: D
147. Indication for lag screw osteosynthesis are all except:
a) Fractures in chin region
b) Fractures of the angle
c) Long sagittal fractures in the body region of mandible
d) Short sagittal fracture in the body region of mandible
Ans: D

Page
91
148. In fracture through menta! foramen in atrophic mandible with more than
treatment would be
a) Champy’s plate b) Lag screw
c) Non-rigid fixation d) Reconstruction plates
Ans: D
149. A transverse fracture of symphysis is treated by all of the following except
a) Two compression plates (2 mm) b) Two lag screws
c) Single miniplate fixation (1.5 mm) d) 2.4 mm reconstruction plate
Ans: C
150. Which of the following provides rigid fixation of the fracture site and heals
by primary intention?
a) Direct transosseous wiring b) Miniplate osteosynthesis without IMF
c) Compression plate osteosynthesis without IMF
d) IMF utilsing arch bars
Ans: C
151. The treatment of choice in comminuted fracture mandible is?
a) Mini plate b) External pins
c) Reconstruction plate with central holes
d) Expansive compression plate with screws
Ans: D
152. Transosseous wiring is an example of
a) IMF with osteosynthesis b) Osteosynthesis without IMF
c) IMF d) None of the above
Ans: A
153. Compression osteosynthesis heats fracture mandible by
a) Primary union without callus formation
b) Primary union with callus formation
c) Secondary union without callus formation
d) Secondary union with callus formation
Ans: A
154. Most common complication in mandibular fracture after surgical
management is?
a) Witche’s chin b) Strictures of soft tissue
c) Infection d) Numbness of area supplied by marginal mandibular
Ans: C
155. Which of the following is true of non-union of fracture?
a) Mobility of fracture site b) Bone loss due to infection
c) Early mobilization d) None of above
Ans: B

Page
92
CARDIOPULMONARY RESUSCITATION

1. Among the following which is th e rescue breathing in adults?


a) 6 times/min b) 18 times/min
c) 12 times/min d) 24 times/min
Ans: C
2. The primary airway hazard for an unconscious patient in a supine position is
a) Tongue obstruction b) Bronchospasm
c) Laryngospasm d) Aspiration
Ans: A
3. In CPR, if one incorrectiy applies pressure over the xiphoid process, the
following may be injured
a) Heart b) Liver
c) Spleen d) Lungs
Ans: B
4. During CPR sternum should be depressed
a) Two inches every 5 seconds b) 2 inches every second
c) 1 inch every 10 seconds d) 3 inches every 5 seconds
Ans: B
5. Of the following which is the first step when initiating cardiopulmonary
resuscitation
a) Establish responsiveness b) To establish an air way
c) A precardial thump d) None of the above
Ans: A
6. Which of the following is true of cardiopulmonary resuscitation
a) Compression to ventilation ratio in two-person CPR is 5:1
b) Compression to ventilation ratio in single-person CPR is 15:2
c) Compression should be 60-80/minute in adults
d) Compressions should be 100/minute in children
e) All of the above
Ans: E
7. Standard airway for Ludwig’s angina
a) Tracheostomy b) Cricothyrotomy
c) Nasal intubation d) Oral intubation
Ans: B
8. In an elective tracheostomy the entry should be made
a) Above the cricoid b) Below the cricoid
c) Through the cricothyroid membrane
d) Laterally below the thyroid cartilage
Ans: B

Page
93
9. Continuation of antibiotic medication in gram-negative soft tissue infections
is required
a) At least 2 days after all clinical signs of infection subside
b) Until acute symptoms of infection subside
c) For a minimum of 7-10 days
d) For a period of 21 days
Ans: C
10. Patient with idiopathic thrombocytopenic purpura (ITP) is most likely to
have which of the following postoperative complications
a) Infection b) Haemorrhage c) Oedema
d) Localised alveolar osteitis (dry socket)
Ans: B
11. One suspected rheumatoid arthritis of the TMJ. Helpful positive laboratory
study would be
a) Haematocrit elevation
b) GIT
c) Increased erythrocyte sedimentation rate
d) Tourniquet test (or) Hess test
Ans: C
12. Among the following which anaerobic organism is frequently responsible
for oral and facial infection?
a) Staphylococcus aureus b) Staphylococcus albus
c) Bacteroides fragilis d) Streptococcus viridans
Ans: C
13. To drain pus from an abscess of the pterygomandibuiar space from an
intraoral approach, the muscle most likely to be incised is the
a) Buccinator b) Temporal
c) Medial pterygoid d) Lateral pterygoid
Ans: A
14. A patient came to dental clinic who has a pulse rate of 72, a respiratory rate
of 15, a BP of 120/80, warm pink extremities and pupils that constrict during
near accommodation. What is the most likely diagnosis?
a) Normal patient b) Acute anxiety syndrome
c) Coronary artery disease d) Myopia
Ans: A
15. At what diastolic pressure do you consider the patient to have significant
hypertension? (mm Hg)
a) 110 b) 90 c) 85 d) 65
Ans: A

Page
94
16. Normal factor VIII levels in the blood is
a) 5.0-6.5 IU/mL b) 3.0-4.5 IU/mL
c) 1.5-2.5 IU/mL d) 0.5-1.5 IU/mL
Ans: D
17. Which of the following agent is of value in the post-operative care of
haemophilic patients?
a) Vitamin K b) Munsell’s solution
c) Aminocaproic acid d) Factor 8 Cryoprecipitate
Ans: C
18. Of the following what would be the WBC range in mild infections
a) 4000-8000 cells/mm3 b) 24,000-30,000 cells/mm3
c) 15,000-20,000 cells/mm3 d) None of the above
Ans: C
19. Which of the following is first manifestation of complete respiratory
obstruction?
a) Pronounced retraction of intercostal and supraclavicular spaces
b) Prolonged expiration
c) Cyanosis
d) None of the above. No changes in the patient
Ans: A
20. All of the following bones contain air sinuses except
a) Frontal b) Nasal c) Sphenoid d) Ethmoid
Ans: B
21. Selective neck dissection includes the following criteria:
a) All Cervical lymph nodes are removed
b) All lymph nodes are removed
c) Some lymph nodes removed by radial neck dissection with preservation of
one or more non-lymphatic structures i.e. SAN, SCM, IJV
d) Additional lymph node and non-lymphatic structure related to
non-lymphatic structure is removed
Ans: C
22. The functional neck dissection the following structures are preserved
a) Sternomastoid muscle, spinal accessory nerve and internal jugular vein
b) Sternomastoid muscle, external jugular vein and internal jugular vein
c) Submandibular gland, thoracic duct and facial artery
d) Strap muscles, facial artery and facial vein
Ans: A
23. A typical protein requirement for a critically ill patient would be
a) 1.5 to 2.0 g/kg per day b) 0.25 to 0.3 g/kg per day
c) 2.5 to 3.5 g/kg per day d) 0.15 to 0.20 g/kg per day
Ans: A

Page
95
24. Repositioning of pre-maxilla is a must before cleft lip and palate surgery as
it
a) supports Base for lip suture b) helps in Feeding
c) helps in proper alignment of permanent central incisors
d) helps in preventing speech defects
Ans: A
25. Which is the best method to counteract severe acidosis following
cardiopulmonary resuscitation?
a) Administration of adrenaline by IV route
b) Administration of sodium bicorbonate intravenously
c) Administration of adrenaline intramuscularly
d) No treatment is necessa because it is self limiting
Ans: B
26. During CPR, the chest compression should be
a) 2 inch/second b) 1 inch/second
c) 2 inch/5 seconds d) 1 inch/5 seconds
Ans: A
27. If efforts in cardiopulmonary resuscitation are effective there will be
a) Constriction of pupils b) Dilatation of pupils
c) Immediate hypertension d) None of the above
Ans: A
28. In external cardiac compressions, the compression relaxation cycle should
be repeated…
a) 100 times per minute b) Twice per second
c) 60 times per minute d) 80 times per minute
Ans: C
29. Which of the following may result in interuptions in cardiac compressions?
a) Little changes in blood flow and blood pressure
b) A reduction of the blood flow and blood pressure to zero
c) Carbon dioxide buildup in the lungs
d) None of the above
Ans: B
30. Among the following which always indicates obstruction to the airway?
a) Increased respiratory rate b) Increased pulse rate
c) Stertorous breathing d) Decreased blood pressure
Ans: C
31. To perform tracheostomy entry should be made at the
a) Cricothyroid ligament b) Thyroid membrane
c) Thyroid notch d) Cricoid cartilage
Ans: A

Page
96
32. Among the following which variable has the greatest significance in
antibiotic management of dental infection
a) Susceptibility of the organism b) Route of administration
c) Antigenicity of the antibiotics d) Specific antibody titer of the host
Ans: A
33. After I & D of an abscess, the infectious process has failed to regress in spite
of the patient being on high doses of an antibiotic, it would be wise to
a) repeat culture and sensitivity tests.
b) insert a large drain.
c) augmenting antibiotic action by administration of parenteral proteolytic
enzymes.
d) debride and irrigate the area with a fibrinolytic agent.
Ans: A
34. After administration of inferior alveolar nerve block to an addict, the
needle accidentally pricks the dentist’s finger. Subsequently the dentist
developed malaise, weakness, and elevated SGOT and SGPT. Most probably he
contacted which of the following diseases?
a) Serum hepatitis b) Infectious hepatitis
c) Infectious mononucleosis d) Primary syphilis
Ans: A
35. After a depressed fracture of zygomatic arch, mandibular movement is
restricted. The most probable reason is
a) Disruption of TMJ
b) Spasm of the lateral pterygoid muscle
c) Mechanical impingement of the fracture fragment on the coronoid process
d) Splinting action of masseter and medial pterygoid muscle
Ans: C
36. Among the following which way best protects from the toxic aspects of a
local anaesthetic?
a) Take a thorough medical history
b) Have oxygen available
c) Use an aspirating technique
d) Use the lowest possible concentration of the local anaesthetics
Ans: C
37. Which of the following is significant in a patient with renal transplant?
a) Haematocrit 37% b) Cholesterol 200 mg%
c) WBC -2000 d) BUN 21%
Ans: D

Page
97
38. Which of the following signs/symptoms does not suggest postoperative
infection?
a) Firm, tender swelling b) Localized pitting oedema
c) Temperature elevation d) Increasing or persisting pain
Ans: B
39. Which of the following values should be taken if there is excessive blood
loss after a surgical procedure?
a) Haematocrit and platelet count b) WBC and RBC count
c) Haemoglobin and Haematocrit d) Platelet and RBC count
Ans: C
40. Which of the following is best treatment for hypoglycaemia in an
unconscious diabetic patient
a) Intravenous administration of 50% dextrose in water
b) Administration of oral carbohydrates
c) Sublingual injection of 50% dextrose
d) None of the above. No treatment is necessary.
Ans: A
41. Near accurate body temperature can be obtained if one measures
a) Orally b) Axillary c) Rectally d) Temperature over forehead
Ans: C
42. Usually acute pyogenic bacterial infection produces
a) Neutropenia b) Lymphocytic leukocytosis
c) Neutrophilic leukocytosis d) Eosinophilia
Ans: C
43. Surgery of the cleft lip repair should be carried out at:
a) 2-5 weeks b) 5-12 weeks
c) 6-12 months d) After I year
Ans: B

Page
98
MISCELLANEOUS

1. During the apicectomy on a maxillary lateral incisor, one should take care
not to perforate (or) damage the
a) Maxillary sinus b) Labial frenum
c) Floor of the nose d) Contents of the incisive canal
Ans: C
2. In sutured skin incisions of the face, the edges of the skin should be
a) Everted b) Inverted
c) Edge to Edge d) Separated
Ans: A
3. Z-plasty is performed in cases of
a) High frenal attachment b) Shaliow sulcus
c) Deep sulcus d) Low frenal attachment
Ans: A
4. Y – V plasty procedure is used for
a) Vestibuioplasty b) Ridge augmentation
c) Frenectomy d) Mucogingival surgery
Ans: C
5. Which of the following criteria should be considered before a third molar
transplantation?
a) The root is atleast half formed
b) The width of the crown approaches the width of the extracted tooth
c) A & d) None of the above
Ans: C
6. Which one of the following disorders responds favorably while using an
occlusal separator
a) Capsular fibrosis b) Muscle spasm
c) Chronic dislocation d) Unilateral condylar hyperplasia
Ans: B
7. Treatment for fibrous dysplasia in a young 25 yr old patient involving
maxilla is best treated by
a) Enbloc resection b) Cosmetic contouring
c) Maxillary resection d) Radiation therapy
Ans: B
8. Irritational fibroma which is asymptomatic is best treated by
a) No treatment is necessary because it is asymptomatic
b) Simple excision
c) Radiation therapy
d) Wide excision followed by chemotherapy
Ans: B

Page
99
9. Minimum volume of local anaesthetic agent is advised for infiltration when
excising frenum because larger volumes may
a) cause tissue slough b) distort the tissues
c) cause rebound bleeding d) cause toxic effects
Ans: B
10. The V plasty procedure is commonly used to correct
a) Oro antral fistula b) Ankyloglossia
c) Interfering labial frenum d) B & C
Ans: D
11. At the time of extraction of upper third molar, the tuberosity is fractured,
but still attached to the periosteum. Which of the following treatment should
be employed?
a) Remove the tuberosity and file the area and suture the soft tissue.
b) Remove the tuberosity, fill the defect with osteogen and suture it
c) Leave the tuberosity and stabilize it
d) None of the above
Ans: C
12. The main disadvantage during excision of tumor involving soft palate is
a) Damage to glossopharyngeal nerve
b) Result in peritonsillar absence
c) Contraction of scar tissue
d) Infection spreading to pterygoid venous plexus
Ans: C
13. After the incisional biopsy of a lesion in the floor of the mouth and
placement of sutures, the patient experiences severe bilateral sweiiing of the
submental and submandibular spaces. This is caused by
a) Ludwig’s infection
b) Too much fluid intake post operatively
c) Injury to lingual artery with haemorrhage
d) Extravasation into these compartments
Ans: D
14. Fibrous dyspiasia can be treated by
a) Irradiation of the lesion b) Surgical excision
c) Conservative surgery d) Removal of the adjacent teeth
Ans: C
15. The best treatment option for an aggressive multicystic ameloblastoma is
a) Curettage to avoid tissue morbidity
b) Marsupialization to avoid damage of adjacent vital structures
c) Enucleation
d) Resection and reconstruction of the resultant defect Ans: D

Page
100
16. Compound odontomas present centrally within the jaws appear in the form
of:
a) Multiple small toothlike structures b) Irregular masses of dentin and enamel
c) No anatomic resemblance to a tooth
d) Non of the above
Ans: A
17. All of the following lesions have a multilocular appearance on the
radiograph except:
a) Odontogenic keratocyst (OKC) b) Amelobalstoma
c) Central AV malformation d) Dentigerous cyst
Ans: D
18. Treatment of torus palatinus in case of denture patients is attained by
a) Irradiation
b) Injection of sclerosing solition
c) Excision of torus and adjacent teeth
d) Excision of torus with chisel and bone file
Ans: D
19. One of the causes of dislocation of TMJ is
a) Yawning b) Epileptic seizures
c) Myositis ossificans d) Filling of carious teeth
Ans: A
20. Ludwig angina affect
a) Submandibular, submental and sublingual spaces unilaterally
b) Submandibular, submental and sublingual spaces bilaterally
c) Submandibular, submental and parapharyngeal spaces bilaterally
d) All of the above
Ans: B
21. Odontogenic keratocyst occure most commonly in
a) Maxillary molar region b) Mandibular anterior region
c) Mandibular premolar-molar area d) Mandibular third molar-ramus area
Ans: D
22. The most common complain of a patient with recent oro-antral
fistula is
a) Pus oozing from the fistula b) Difficulty of smoking
c) Regurgitation of fluids from nose d) Severe throbbing pain
Ans: C

Page
101
23. The following are the anatomical features which obstruct the flow
of maxillary sinus drainage except:
a) Polyps b) Thickened lining
c) Middle concha d) Higher level of natural opening
Ans: C
24. Mucocele of the lower lip is treated by:
a) Incision and drainage b) Enucleation c) marsupialization
d) excision including the adjacent minor salivary gland
e) marsupialization followed by enuceation
Ans: D
25. Inability to open the mouth and masticate food is due to
a) Facial deformity b) Ankylosis of TMJ
c) Myofacial pain dysfunction syndrome d) Periapical periodontitis
Ans: B
26. Alveolotomy is
a) Cutting into the alveolar process
b) Surgical excision of the alveolar process
c) Surgical contouring of the alveolar process
d) Surgical contouring of the gingival tissues
Ans: A
27. Osteitis deformans is the condition in which
a) Roots have hypercementosis b) Anodontia seen
c) Supernumerary roots are more common d) None of the above
Ans: A
28. Bifid mandibular canal is somewhat common in the following disease
a) Paget’s disease b) Fibrous dysplasia
c) Neurofibromatosis d) A & B
Ans: C
29. Which of the following is a constant finding in systemic infection?
a) Fever b) Swelling c) Bacteremia d) Lymphadenopathy
Ans: A

Thank you

Page
102

Potrebbero piacerti anche